Anda di halaman 1dari 44

PSIQUIATRÍA

Principios generales 5% - 10
Sistemas de órganos 90% - 95%
- Trastornos mentales 75% - 85
o Promoción de la salud y mantenimiento de la salud 1% - 5%
o Comprensión de los mecanismos de la enfermedad 10% - 15
o Establecer un diagnóstico 55% - 65
 Los trastornos mentales suelen diagnosticarsepor primera vez en la infancia, niñez o
adolescencia (5-10%).
 Trastornos relacionados con sustancias (5-10%)
 Esquizofrenia y otros trastornos psicóticos (5-10%)
 Trastornos del estado de ánimo (5-10%)
 Trastornos de ansiedad (5-10%)
 Trastornos somatomorfos (1-5%)
 Otros trastornos/condiciones (5-10%)
o Aplicación de los principios de gestión 20-25
- Enfermedades del sistema nervioso y de los sentidos especiales 10%-15

Sesión de revisión:
- MUCHAS preguntas sobre el abuso de sustancias
- Depresión, estado de ánimo, ansiedad
o Suicidio: factores de riesgo
 Cuando el paciente puede nombrar algo que ha cambiado, estás en mejor
posición que cuando no puede hacerlo.
 Cambio agudo de trabajo, inutilidad, desesperanza, abuso de sustancias,
ideación suicida, intento, plan, varones caucásicos de edad avanzada, escaso
apoyo familiar, antecedentes de suicidio, familiares con antecedentes de
suicidio.
- El trastorno dismórfico corporal es un tema de 1 pregunta
- Preguntas infantiles - alrededor del 25% son niños/adolescentes como paciente. (no significa
trastornos infantiles - podría ser MDD, anorexia)
- No hay teóricos
- Habrá cosas de psicoterapia - no habrá freud ni teorías, ni mecanismos de defensa
- Todo se probará con diagnósticos DSM.
- Esquizofrenia - progresión - breve, esquizofreniforme, esquizofrenia

Neuro:
- demencias
- RM y presentaciones infantiles de la RM, pronóstico

Una pregunta desórdenes:


- Somatomorfo - trastorno de somatización, conversión d/o
o Conversión con personas OC - pérdida de fxn - tx con terapia adecuada (PT para mal
movimiento del brazo, logopeda para mal hablar, etc).
o Hipocondriasis - con antenas - tranquilizadora (basada en la ansiedad y puede tratarse
con ISRS en la vida real.
o Somatización -
o Trastornos dismórficos corporales
- Trastornos de la alimentación - La bulimia puede tratarse con SRIS, la anorexia con tratamiento
conductual
- Trastornos del sueño
- Trastorno de dolor
- trastornos disociativos
Puntos clave:
- Trastorno de adaptación vs. TDM vs. TAG vs. Trastorno de estrés agudo vs. TEPT
- Medicamentos, ESPECIALMENTE efectos secundarios
- Trastornos médicos con manifestaciones psiquiátricas
o Depresión y cáncer de páncreas, enfermedad de Addison
o Feocromocitoma o carcinoide con aspecto de trastorno de pánico
- Depresión vs. duelo vs. trastorno de somatización
- Características físicas de las presentaciones comunes de la RM (síndrome de Down, X frágil)
- Buscar delirios y psicosis
- Medicamentos Efectos secundarios
o A nivel del receptor
- Neurotransmisores
o Afectados en enfermedades psiquiátricas
 Ansiedad - aumento de NE, disminución de GABA y serotonina
 Esquizofrenia - serotonina elevada, NE elevada, GABA disminuido, exceso de dopamina
en la corteza prefrontal = sx negativo. Dopamina en la vía mesolímbica = síntomas
positivos
 Bloqueo de la dopamina en Tubuloinfundibular - hiperprolactinemia;
Nigrostriatal - EPS
 Depresión - disminución de serotonina, NE, dopamina; cortisol elevado, eje tiroideo
anormal.
 Trastorno de pánico - aumento de NE, disminución de GABA, serotonina
 Demencia de Alzheimer: disminución de Ach debido a la pérdida de neuronas
colinérgicas en el núcleo basal de Meynert en el mesencéfalo y disminución de NE
debido a la pérdida de neuronas noradrenérgicas en el locus ceruleus del tronco
encefálico.
 Enfermedad de Parkinson: pérdida de células en la sustancia negra de los ganglios
basales, lo que provoca una disminución de la dopamina y la pérdida de tractos
dopaminérgicos.
 Tourette/Tics: alteración de la regularización de la dopamina en el núcleo caudado y
posible regularización de los opiáceos endógenos y del sistema noradrenérgico.
o Lugar de liberación de NT
 Locus ceruleus - NE
 Núcleo de Raphe - serotonina
- Intoxicación y síndrome de abstinencia Sección de primeros auxilios

TRASTORNOS PSICÓTICOS
Esquizofrenia
Para hacer el diagnóstico de esquizofrenia, el paciente debe tener síntomas de la enfermedad durante al menos
_____.
- 6 meses
Nombra los 5 subtipos de esquizofrenia.
- Paranoico
- Desorganizados
- Catatónico
- Indiferenciado
- residual
¿Cuáles son las 5 características de la esquizofrenia catatónica?
- Inmovilidad motriz
- excesiva actividad motora sin propósito
- negativismo extremo o mutismo
- movimientos voluntarios o posturas peculiares
- ecolalia/ecopraxia
¿Cuáles son las "5 A" de la esquizofrenia, que describen los síntomas negativos de la enfermedad?
- Anhedonia
- Afecto (plano)
- Alogia (pobreza de palabra)
- Avolición (apatía)
- Atención (escasa)
¿Cuáles son los 5 síntomas que deben estar presentes durante al menos 1 mes para diagnosticar esquizofrenia?
- Delirios
- Alucinaciones
- discurso desorganizado
- comportamiento extremadamente desorganizado o catatónico
- síntomas negativos
Diferenciar entre trastorno psicótico breve, trastorno esquizofreniforme y esquizofrenia en función de la duración de
los síntomas.
- Trastorno psicótico breve: < 1 mes
- trastorno esquizofreniforme: 1 - 6 meses
- esquizofrenia: > 6 meses
¿Qué son las "ideas de referencia"?
- Creencia de que las señales del entorno externo están relacionadas únicamente con el individuo
¿En qué consiste la hipótesis de la deriva descendente con respecto a los esquizofrénicos?
- Las personas que padecen esquizofrenia son incapaces de desenvolverse bien en la sociedad y, por tanto,
entran a formar parte de los grupos socioeconómicos más bajos
¿Cuáles son las dos vías dopaminérgicas teóricamente afectadas en la esquizofrenia?
- Cortical prefrontal: actividad dopaminérgica inadecuada responsable de los síntomas negativos.
- mesolímbico: actividad dopaminérgica excesiva responsable de los síntomas positivos
¿Cuáles son los 4 componentes de la vía mesolímbica responsables de los sx positivos de la esquizofrenia?
- Núcleo accumbens
- Fornix
- Amígdala
- hipocampo
Se cree que los sx negativos de la esquizofrenia se deben a la acción de la dopamina en la vía _____.
- Mesocortical
Describir los niveles relativos de receptores de serotonina, norepinefrina, GABA y glutamato en pacientes
esquizofrénicos.
- Elevado: serotonina, norepinefrina
- disminuido: GABA, receptores de glutamato
Nombre 4 medicamentos antipsicóticos de primera generación utilizados para tratar la esquizofrenia.
- Clorpromazina
- Tioridazina
- Trifluoperazina
- haloperidol
Los antipsicóticos de primera generación son todos antagonistas de _____.
- Dopamina (principalmente D2)
¿Cuáles son los 4 antipsicóticos de segunda generación utilizados para tratar la esquizofrenia?
- Risperidona; clozapina; olanzapina; quetiapina; aripiprazol; ziprosidona
Los antipsicóticos de segunda generación antagonizan los receptores _____ y los receptores dopaminérgicos.
- Serotonina
¿Cuáles son los efectos secundarios característicos de los antipsicóticos de primera y segunda generación?
- Primera generación: EPS; segunda generación: síndrome metabólico
¿Qué 2 medicamentos antiparkinsonianos pueden utilizarse para tratar el EPS?
- Benztropina
- difenhidramina
¿Qué bloqueo de la vía dopaminérgica por la medicación neuroléptica produce efectos secundarios
extrapiramidales?
- Nigroestriado
¿En qué 3 situaciones es necesario un solo sx psicótico para el diagnóstico de esquizofrenia?
- Delirios extraños
- alucinaciones auditivas comentadas por el paciente
- dos o más voces que hablan entre sí

Trastorno esquizoafectivo
¿Qué criterios se utilizan para distinguir el trastorno esquizoafectivo del trastorno del estado de ánimo con rasgos
psicóticos?
- Delirios o alucinaciones durante 2 semanas en ausencia de síntomas de trastorno del estado de ánimo

Trastorno esquizofreniforme
Psicosis Reactiva Breve

TRASTORNOS DEL ESTADO DE ÁNIMO


Trastorno bipolar
Manía:
Describa la nemotecnia DIG FAST para los síntomas de manía.
- Distracción
- Insomnio/Comportamiento impulsivo
- Grandiosidad
- Vuelo de ideas/pensamientos acelerados
- Actividad/agitación
- Discurso (presionado)
- Irreflexión
¿Qué criterio se requiere para diagnosticar un episodio maníaco?
- Presencia de un estado de ánimo anormalmente elevado, expansivo o irritable y 3 de los síntomas anteriores
que duren al menos 1 semana.
¿Qué 3 fármacos están aprobados para el tratamiento agudo de la manía?
- Litio
- Valproato
- carbamazepina
¿Qué fármaco se utiliza como tratamiento de mantenimiento del trastorno bipolar?
- Lamotrigina
Todos los antipsicóticos de segunda generación están aprobados para tratar la manía aguda, excepto _____.
- Clozapina
¿Qué es el especificador de ciclo rápido?
- Al menos 4 episodios depresivos mayores, maníacos, hipomaníacos o mixtos en los últimos 12 meses.
¿Qué periodo de tiempo se requiere para el diagnóstico de manía frente a hipomanía?
- Manía: 7 días
- hipomanía: 4 días
¿Cuál es la definición de ciclismo rápido?
- La aparición de 4 o más episodios del estado de ánimo en 1 año (depresivo mayor, maníaco, mixto, etc.).
(T/F) El trastorno bipolar I tiene el mayor vínculo genético de todos los trastornos psiquiátricos importantes.
- Verdadero
¿Cuál es el mejor tratamiento para una mujer maníaca durante el embarazo?
- TEC
¿Qué 2 anticonvulsivos son especialmente útiles para el trastorno bipolar de ciclo rápido?
- Carbamazepina; ácido valproico
Depresión bipolar
Estabilizador del ánimo.

Ciclotimia
Trastornos depresivos
Duelo
¿Qué periodo de tiempo significa la transición de un duelo normal a uno complicado o prolongado?
- 6 meses
(T/F) El duelo normal puede incluir desorganización grave o suicidio.
- Falso

Depresión posparto
Describa el especificador de inicio posparto.
- Episodio depresivo, maníaco o mixto en las primeras 4 semanas postparto
(T/F) La depresión posparto suele resolverse sin medicación.
- Verdadero
¿Qué momento de inicio se requiere para el especificador de depresión posparto?
- Inicio a las 4 semanas del parto

Trastorno afectivo estacional


¿Cuál es la tríada del trastorno afectivo estacional?
- Irritabilidad
- antojo de carbohidratos
- hipersomnia

Trastorno depresivo mayor


¿Cuál es la tasa de respuesta de los pacientes con depresión mayor a los antidepresivos?
- El 70% responderá; el 30% de esta respuesta es un efecto placebo
- tardan de 4 a 8 semanas en hacer efecto (hasta 3 meses en pacientes de edad avanzada)
¿Qué 2 características definen la depresión mayor?
- Estado de ánimo depresivo; pérdida de interés o placer
La mayoría de los episodios depresivos desaparecen espontáneamente en _____.
- 6 meses
Nombra 4 subcategorías únicas de trastornos depresivos.
- Melancólico - anhedonia, despertares tempranos, PMR, culpa excesiva, anorexia
- Atípico: hipersomnia, hiperfagia, humor reactivo, parálisis plomiza, hipersensibilidad al rechazo
interpersonal.
- Catatónico: catalepsia (inmovilidad), actividad motora sin propósito, negativismo o mutismo extremo,
posturas extrañas, ecolalia; tratamiento con antidepresivos y antipsicóticos simultáneamente.
- psicótico - delirios/alucinaciones
Mencione las 2 características necesarias para el diagnóstico de la melancolía.
- Pérdida generalizada de interés o placer; incapacidad para responder a estímulos placenteros.
¿Cuáles son las 2 únicas características descriptivas de la melancolía?
- Variación diurna; insomnio terminal
¿Cuáles son las 5 características de la depresión atípica?
- Hipersomnia
- Hiperfagia
- estado de ánimo reactivo
- parálisis de plomo
- hipersensibilidad al rechazo interpersonal
¿Qué categoría de antidepresivos ha demostrado ser especialmente eficaz en el tratamiento de la depresión atípica?
- Inhibidores de la monoaminooxidasa (IMAO)
Nombra 3 cambios en el sueño con la depresión.
- Disminución del sueño delta; disminución de la latencia REM; aumento de la densidad REM
Describir la progresión de los ensayos de fármacos antidepresivos.
- Los ensayos con fármacos deben durar entre 4 y 8 semanas; si el paciente no responde en 4 semanas de
tratamiento, debe aumentarse la dosis o cambiarse a otro fármaco, preferiblemente de otra clase.
La potenciación de los fármacos antidepresivos con _____ es la opción mejor investigada.
- Carbonato de litio
¿Qué medicamento antipsicótico de segunda generación se utiliza a menudo como antidepresivo de aumento?
- Aripiprazol
_____ tiene la tasa de suicidio más alta de todos los trastornos psiquiátricos.
- MDD
¿Qué escala se utiliza como medida estándar de la gravedad de la depresión en la investigación para evaluar la
eficacia de las terapias?
- Puntuación de Hamilton
¿Qué cáncer está más relacionado con la depresión?
- Cáncer de páncreas
El uso de IMAO es a/w ¿qué 2 condiciones adversas?
- Crisis hipertensiva
- síndrome de la serotonina
(T/F) El tratamiento del TDM con rasgos psicóticos puede realizarse con un antidepresivo solo.
- Falso; el tratamiento debe incluir un antidepresivo, así como un medicamento antipsicótico o terapia
electroconvulsiva.
¿Cuál es el protocolo para los cambios de medicación en un niño con depresión mayor con rasgos psicóticos que
responde a la combinación de un antidepresivo y un antipsicótico?
- El antipsicótico atípico debe continuarse durante 3 meses y luego reducirse; el antidepresivo debe
continuarse durante 6 - 9 meses y luego reducirse durante 2 - 3 meses al 33% por mes.
Describir los resultados de un escáner PET en un paciente con depresión.
- Reducción de la actividad metabólica y del flujo sanguíneo en ambos lóbulos frontales.
El uso de tetraciclina, nifedipino y verapamilo es a/w _____.
- Depresión

El tratamiento de la depresión mayor en adultos mayores tiene más éxito con una combinación de
medicamentosantidepresivos más psicoterapia interpersonal de apoyo. La psicoterapia interpersonal es
adecuada para personas que han experimentado conflictos interpersonales recientes o transiciones de roles difíciles.

Trastorno distímico
¿En qué difiere el diagnóstico del trastorno distímico entre niños y adultos?
- El trastorno distímico puede diagnosticarse en niños si presentan síntomas durante un periodo de tiempo de
1 año (en lugar de los 2 años necesarios para los adultos).

Terapia electroconvulsiva
Nombra 4 indicaciones para la terapia electroconvulsiva (TEC).
- Depresión muy grave
- alto potencial de suicidio
- enfermedades cardiovasculares
- embarazo
- no responde a la farmacoterapia
Describa 2 características del TDM en las que estaría indicado el tratamiento con TEC.
- TDM con rasgos psicóticos
- MDD cuando se requiere una respuesta rápida
La eficacia de la TEC se basa en _____, no en _____.
- Duración de la supresión postictal; duración de la crisis
Método:
- premedicación con atropina  anestesia generalizada  relajante muscular  convulsión generalizada
por corriente a través del cerebro (bilateral/unilateral)  convulsión < 1 minuto.
- 8 tx en un periodo de 2-3 semanas.
- Amnesia retrógrada - SE común pero desaparece en 6 meses.
NO existen contraindicaciones reales - el aumento de la presión intracraneal es la más perjudicial/asociada. El IM
reciente no es una contraindicación absoluta.

TRASTORNOS DE ANSIEDAD
Trastorno de pánico con y sin agorafobia
¿Cuál es el rasgo distintivo del trastorno de pánico?
- Ataques de pánico inesperados no provocados por ningún estímulo en particular.
¿Qué dos elementos debe incluir el régimen de tratamiento del trastorno de pánico?
- Antidepresivos; TCC
¿Qué otro trastorno se observa con frecuencia en pacientes con trastorno de pánico?
- MDD
Nombra 5 afecciones médicas relacionadas con los ataques de pánico.
- Prolapso de la válvula mitral
- Asma
- émbolo pulmonar
- angina
- anafilaxia
Mencione 3 características del trastorno de pánico.
- Preocupación persistente por sufrir nuevos ataques
- preocuparse por las consecuencias del atentado
- cambio significativo en el comportamiento relacionado con los ataques
¿Qué especificador se requiere con el diagnóstico de trastorno de pánico?
- Con o sin agorafobia
¿Qué 2 ISRS se utilizan habitualmente para tratar el trastorno de pánico?
- Paroxetina
- sertralina
Nada trata un ataque de pánico como una BZD - pero no empieces - ¿persona que se hace adicta a las benzos o no?

Fobias específicas y sociales


¿Qué calificativo se requiere para el diagnóstico de fobias en pacientes menores de 18 años?
- Duración > 6 meses
¿Qué ISRS está aprobado por la FDA para el tratamiento de la fobia social?
- Paroxetina
Fobia social: desensibilización gradual; propranolol: para las manifestaciones físicas; no se ocupa de la ansiedad
real.
Fobia escolar en los niños - inundaciones

Los cuatro trastornos mentales más comunes:


1. fobias
2. trastornos inducidos por sustancias
3. depresión grave
4. TOC

Trastornos obsesivo-compulsivos
¿Qué TCA ha demostrado ser eficaz en el tratamiento del TOC?
- Clomipramina
Diferenciar entre obsesión y compulsión.
- Obsesión: pensamiento, sentimiento o idea egodistónica recurrente e intrusiva.
- Compulsión: comportamiento repetitivo consciente vinculado a una obsesión que, cuando se realiza,
funciona para aliviar la ansiedad causada por la obsesión.
La tasa de TOC es mayor en pacientes con familiares de primer grado que tienen _____.
- Síndrome de Tourette

Trastorno de estrés postraumático


¿Qué 2 clases de medicamentos deben utilizarse para tratar el TEPT?
- ISRS; agonistas alfa-2 (p. ej., prazosina, clonidina)
Pesadillas, flashbacks, respuestas de sobresalto
El tratamiento es HABLAR: la psicoterapia psicoanalítica presta especial atención al papel que desempeñan el
trauma y las experiencias sexuales en la configuración tanto del contenido inconsciente como del
procesamiento inconsciente.
Trastorno de ansiedad generalizada
¿Qué antidepresivo atípico se utiliza para el tratamiento del trastorno de ansiedad generalizada?
- Buspirona
Describir la indicación y el mecanismo de la buspirona.
- Ansiolítico no sedante no benzodiacepínico aprobado por la FDA para el tratamiento de los trastornos de
ansiedad.
- actúa como agonista de los receptores 5HT-1A
¿Qué 4 medicamentos/clases de medicamentos se utilizan en el tratamiento del TAG?
- ISRS; venlafaxina; buspirona; benzodiacepinas
(T/F) La buspirona no es tan eficaz en pacientes ya expuestos a las benzodiacepinas.
- verdadero
Describir la acción ansiolítica de la buspirona.
- Agonista parcial del receptor 5HT-1A
¿Cuál es la ventaja de la buspirona sobre las benzodiacepinas?
- No potencia la depresión del SNC del alcohol (útil en alcohólicos) y tiene un bajo potencial de
abuso/adicción
¿Qué medicamento es útil para los pacientes que desean una medicación de acción rápida y corta duración, pero que
no toleran las benzodiacepinas por diversos motivos?
- Hidroxizina (antihistamínico)
Tratamiento:
- Psicoterapia

PSIQUIATRÍA INFANTIL
Retraso mental/Inteligencia/cognición
¿Cuál es el test de CI más utilizado para pts de 16 a 75 años?
- Escala Wechsler de inteligencia para adultos (WAIS)
¿Qué test de CI se utiliza en pts de 2 a 18 años?
- Prueba Stanford-Binet
¿Qué 2 tests de inteligencia se utilizan para niños y adolescentes?
- De 2 a 12 años: Batería de evaluación Kaufman para niños (K-ABC)
- 6-16 años: Escala Wechsler de Inteligencia para Niños-Revisada (WISC-R)
¿Qué 3 características son necesarias para diagnosticar un retraso mental?
- CI < 70
- déficits en las habilidades de adaptación apropiadas para el grupo de edad
- aparición antes de los 18 años
Nombra y describe los niveles de CI que corresponden a las 4 categorías de retraso mental.
- Suave: 70-55
- moderado: 55-40
- grave: 40-25
- profundo: < 25
¿Cuál es la forma hereditaria más común de retraso mental? ¿Cuál es su causa?
- Síndrome del cromosoma X frágil, causado por un defecto en el gen FMR-1
¿Cuál es el trastorno del aprendizaje más frecuente?
- Trastorno de la lectura, que afecta hasta al 10% de los niños en edad escolar.
Nombra 4 trastornos generalizados del desarrollo (TGD).
- Trastorno autista
- Trastorno de Asperger
- Trastorno de Rett
- trastorno desintegrativo infantil

Síndrome de Noonan - Una niña de 8 años es llevada a su consulta después de que su madre notara que no "crecía
bien". La madre afirma que la niña saca malas notas en el colegio, coge infecciones de "catarro y tos" muy a menudo
y se hace moratones con mucha facilidad. Observa que la niña tiene cara triangular, hipertelorismo, surco nasolabial
profundo y ojos rasgados. También se observa la presencia de cuello palmeado y orejas de implantación baja. La
niña tiene escoliosis y un coeficiente intelectual de 65. A continuación, solicita una analítica completa que incluya
un hemograma, un perfil de coagulación y una evaluación cardiaca. La ecocardiografía detecta la estenosis de la
válvula pulmonar. Ante la sospecha de un trastorno congénito, solicita un cariotipo y un análisis de mutaciones.
El diagnóstico en esta niña es síndrome de Noonan (SN). Las mutaciones del gen PTPN11se observan en el 50% de
los individuos con síndrome nefrótico. Las mutaciones de SOS1y KRAS son otras causas, aunque la ausencia de una
mutación no descarta el síndrome nefrótico.El síndrome nefrótico es un trastorno congénito esporádico o autosómico
dominante con rasgos fenotípicos típicos, que pueden no ser visibles para el observador casual. Los rasgos faciales
más comunes incluyen hipertelorismo y orejas de implantación baja y rotadas hacia atrás, junto con un grueso
hélix. El surco nasolabial está profundamente estriado en más del 90% de los casos. Otras características son
defectos cardíacos congénitos, trastornos hemorrágicos, retraso mental, cuello palmeado y baja estatura. La
estenosis de la válvula pulmonar es el defecto cardíaco más frecuente y se observa en el 50% de los afectados.
Otros hallazgos cardíacos incluyen defectos septales auriculares y/o ventriculares y miocardiopatías.

El síndrome de Mowat-Wilson es una enfermedad autosómica dominante que resulta de la mutación o deleción del
gen ZEB2 en el cromosoma 22. Los rasgos característicos son: mentón estrecho; ojos profundos y muy
separados; boca abierta; lóbulos de las orejas levantados; puente nasal ancho; y surco nasolabial acortado.
También se observan retraso mental, retraso del crecimiento y del desarrollo motor, cardiopatías y enfermedad de
Hirschsprung.

La mutación en el gen del retraso mental frágil 1(FMR1) causa el síndrome del cromosoma X frágil (FXS). El FXS
es la causa más común de retraso mental hereditario y se caracteriza por problemas cognitivos, neuropsicológicos y
de comportamiento. Se observa un comportamiento similar al autismo, retrasos en el habla y el lenguaje, ansiedad,
retraso mental (CI 35-70), timidez, trastorno por déficit de atención con o sin hiperactividad y facies típica.

El gen FBN1codifica la proteína fibrilina-1, y una mutación en este gen causa el síndrome de Marfan. Otras
afecciones que pueden derivarse de la mutación son la ectopia lentis aislada, el síndrome MASS, etc.La mutación en
el gen KAL del cromosoma 22 es la causa más común del síndrome de Kalmann. Produce deficiencia de la hormona
liberadora de gonadotropina junto con anosmia o hiposomía grave debida a hipoplasia o agenesia del bulbo
olfatorio. Los hallazgos incluyen ausencia de desarrollo sexual secundario, anosmia, sordera neurosensorial,
trastornos convulsivos, disminución de la densidad ósea, metacarpos cortos e infertilidad.

X frágil
La respuesta correcta es síndrome X frágil (FXS).El FXS es la causa más común de retraso mental hereditario y se
caracteriza por problemas cognitivos, neuropsicológicos y de comportamiento. Se observa un comportamiento
similar al autismo, retrasos en el habla y el lenguaje, ansiedad, retraso mental (CI 35-70), timidez, trastorno por
déficit de atención con o sin hiperactividad y facies típica. El síndrome es el resultado de una repetición CGG
expandida (> 200 repeticiones) en la porción no traducida 5' del gen del retraso mental frágil 1(FMR1). Esto provoca
una deficiencia (o ausencia) de la proteína FMR. La proteína FMR controla la traducción de varios genes que
regulan la plasticidad sináptica y el desarrollo.

El síndrome de Marfan es un trastorno genético que afecta al tejido conjuntivo del cuerpo y está causado por
mutaciones en el gen FBN1 del cromosoma 15. Los individuos afectados tienen una estatura superior a la media y
pueden presentar subluxación del cristalino, disección aórtica y neumotórax, aparte de otras anomalías.

El síndrome de Williams es una enfermedad genética rara conrasgos característicos "elfos", defectos
cardiovasculares, hipercalcemia y defectos del neurodesarrollo. El retraso mental suele ser de leve a moderado.
Por lo general, la adquisición del lenguaje se retrasa y, aunque la calidad del lenguaje es algo normal, el deterioro
del lenguaje está presente a lo largo de toda la vida. Otros rasgos característicos son los déficits visuoespaciales y la
hipersociabilidad. El hipotiroidismo, el retraso del crecimiento y las dificultades de alimentación son características
adicionales. La mayoría de los casos son esporádicos, y los casos hereditarios son en su mayoría autosómicos
dominantes.

El síndrome XYY es una enfermedad en la que los varones tienen un cromosoma Y de más; se caracteriza por una
mayor estatura, retrasos en el habla y dificultades de aprendizaje.

El síndrome de Noonan es un trastorno congénito esporádico o autosómico dominante con rasgos fenotípicos
típicos, que pueden no ser visibles para el observador casual. Los rasgos faciales más comunes incluyen
hipertelorismo y orejas de implantación baja y rotadas hacia atrás, junto con un grueso hélix. El surco
nasolabial está profundamente estriado en más del 90% de los casos. Otras características son defectos cardíacos
congénitos, trastornos hemorrágicos, retraso mental, cuello palmeado y baja estatura.

Trastorno autista
Describa las 3 categorías de síntomas utilizadas para diagnosticar el autismo.
- Problemas con la interacción social; deficiencias en la comunicación; patrones repetitivos/estereotipados de
comportamiento o actividades.
¿Cuál es la edad requerida para el autismo?
- 3 años
(T/F) El 70% de las personas con autismo cumplen los criterios de retraso mental.
- Verdadero

Trastorno de Asperger
Trastorno de Rett
¿Cuáles son las características del trastorno de Rett?
- Desarrollo físico y psicomotor normal durante los primeros 5 meses después del nacimiento, seguido de
una disminución del ritmo de crecimiento de la cabeza y de la pérdida de habilidades manuales
previamente aprendidas entre los 5 y los 30 meses de edad.
- los niños desarrollan entonces movimientos estereotipados de las manos (retorcerse/lavarse las manos),
trastornos del lenguaje y retraso psicomotor, y problemas con la marcha o los movimientos del tronco
¿Con qué mutación genética se asocia el trastorno de Rett?
- Mutación del gen MECP2 en el cromosoma X

Trastorno por déficit de atención e hiperactividad (TDAH)


¿Cuáles son las 3 subcategorías del TDAH?
- Predominantemente desatento
- predominantemente hiperactivo-impulsivo
- tipo combinado
Los niños con TDAH tienen una probabilidad del _____ de desarrollar un trastorno antisocial de la personalidad.
- 25%
¿Cuáles son las 3 indicaciones clínicas para el uso de atomoxetina en el tratamiento del TDAH?
- Individuos o familias con problemas de abuso de sustancias; individuos con tics; pacientes con trastornos
de ansiedad comórbidos.
¿Qué metal pesado puede provocar hiperactividad que se disfraza de TDAH?
- Plomo
Nombra 3 medicamentos utilizados para tratar el TDAH.
- Dextroanfetamina; metilfenidato; atomoxetina
¿Qué es la atomoxetina? (Strattera)
- Un no estimulante aprobado por la FDA para el tratamiento del TDAH
¿Qué clase de fármaco se utiliza para tratar el TDAH en caso de que fallen los estimulantes del SNC?
- Agonistas alfa-2 (clonidina, guanfacina)
Trastorno desintegrativo infantil
¿Cuál es el intervalo de edad para el desarrollo del trastorno desintegrativo infantil?
- De 2 a 10 años

Eliminación
¿Cuáles son los 2 criterios para el diagnóstico de la encoporesis?
- Más de 5 años
- ya han aprendido a ir al baño
Define enuresis.
- Vaciado involuntario de orina (enuresis) después de los 5 años (al menos dos veces por semana durante al
menos 3 meses consecutivos o con deterioro marcado).
¿Qué TCA puede utilizarse para tratar la enuresis?
- Imipramina
Trastorno negativista desafiante vs. Trastorno de conducta
¿En qué se diferencia el trastorno negativista desafiante (TOD) del trastorno de conducta?
- El TOD no implica agresión física ni violación de los derechos básicos de los demás.
Describa las 4 categorías de comportamientos descritos por el trastorno de conducta.
- Agresión hacia personas o animales
- destrucción de bienes
- engaño/robo
- violación grave de las normas
¿Cuál es el periodo de diagnóstico entre el TOD y el trastorno de conducta?
- ODD: 6 meses
- trastorno de conducta: 12 meses

Trastorno de Tourette
Describir el componente temporal de los tics necesario para el diagnóstico del trastorno de Tourette.
- Los tics ocurren varias veces al día, casi todos los días durante > 1 año (ningún período sin tics > 3 meses)
(T/F) Sólo se requieren tics motores o vocales para el diagnóstico del trastorno de Tourette.
- Falso; tanto los tics motores como los vocales deben estar presentes para diagnosticar el trastorno de
Tourette.
Describir las bases neuroquímicas del síndrome de Tourette.
- Deterioro de la regulación de la dopamina en el núcleo caudado
Describa la nemotecnia PANDAS y el trastorno asociado.
- Trastornos neuropsiquiátricos pediátricos autoinmunes asociados a infecciones estreptocócicas
- aparición rápida de TOC y/o trastornos de tics, como el síndrome de Tourette, tras infecciones por
estreptococos betahemolíticos del grupo A
¿Qué 2 clases de medicamentos se utilizan para tratar el trastorno de Tourette?
- Neurolépticos atípicos (por ejemplo, risperidona)
- agonistas alfa-2 (por ejemplo, clonidina, guanfacina)
¿Qué 2 trastornos suelen estar presentes en pacientes con síndrome de Tourette?
- TDAH; TOC

Trastorno de ansiedad por separación


¿En qué franja de edad es apropiada la ansiedad por separación?
- De 7 meses a 6 años

TRASTORNOS ALIMENTARIOS
Anorexia nerviosa
Nombra las 2 categorías de anorexia nerviosa.
- Tipo restrictivo
- tipo de atracón/purga
Describa los requisitos de peso enumerados por el DSM-IV para el diagnóstico de la anorexia nerviosa.
- < 85% de peso corporal ideal o IMC < 17,5 kg/m2
¿Qué alteración ácido-base se encuentra a menudo en los pacientes anoréxicos de tipo atracón/purga?
- Alcalosis hipoclorémica, hipopotasémica
¿Qué característica específica debe estar presente para diagnosticar anorexia nerviosa en mujeres?
- Amenorrea (ausencia de al menos 3 ciclos menstruales consecutivos)

Bulimia nerviosa
Una cuarta parte de los pacientes con bulimia nerviosa tienen _____ comórbido.
- Cleptomanía
¿Qué componente temporal se requiere para el diagnóstico de la bulimia nerviosa?
- Los atracones y las conductas compensatorias se producen al menos dos veces por semana durante 3 meses
Según el DSM-IV, ¿cuál es la característica más diagnóstica de la bulimia nerviosa?
- Episodios recurrentes de atracones
¿Cuál es el mecanismo de la sibutramina en el tratamiento del trastorno por atracón?
- Inhibe la recaptación de norepinefrina, serotonina y dopamina
Describir el componente temporal necesario para la bulimia nerviosa.
- Tanto los atracones como las conductas compensatorias inapropiadas se producen, de media, al menos dos
veces por semana durante 3 meses
¿Cuál es el único ISRS aprobado por la FDA para el tratamiento de la bulimia nerviosa?
- Fluoxetina

TRASTORNOS SOMATOMORFOS
Nombra 5 tipos de trastornos somatomorfos.
- Trastorno de somatización
- trastorno de conversión
- trastorno del dolor
- trastorno dismórfico corporal
- hipocondría
Describa los 8 síntomas necesarios para el diagnóstico del trastorno somatomorfo.
- 4 dolor; 2 gastrointestinal; 1 sexual; 1 pseudoneurológico
El trastorno somatomorfo suele presentarse antes de los _____ años de edad.
- 30

Trastorno de somatización
¿Qué edad de inicio se requiere para el diagnóstico del trastorno de somatización?
- Inicio antes de los 30 años
¿Qué 8 síntomas son necesarios para el trastorno de somatización?
- 4 síntomas de dolor; 2 síntomas gastrointestinales; 1 síntoma sexual/reproductivo; 1 síntoma
pseudoneurológico, no limitado al dolor.

Trastorno de conversión
Define globus hystericus.
- La sensación de nudo en la garganta que experimentan muchos pacientes con trastorno de conversión.

Trastorno de dolor
¿Cuáles son los 2 subtipos de trastorno por dolor?
- Dolor y factores psicológicos
- dolor debido a factores psicológicos y a una afección médica general

Dios dismórfico corporal


Hipocondriasis
Se ha observado que una forma particular de hipocondría, la fobia a la enfermedad, responde a _____.
- Imipramina
¿Cuál es el tratamiento de elección para el trastorno dismórfico corporal y la hipocondría?
- ISRS
(T/F) La hipocondriasis es el único trastorno somatomorfo que no presenta una mayor frecuencia en las mujeres.
- Verdadero

Trastorno facticio frente a falsedad


¿Qué distingue el trastorno facticio del fingimiento?
- El trastorno facticio carece de ganancia secundaria

TRASTORNOS DE LA PERSONALIDAD
Los trastornos de la personalidad no pueden diagnosticarse antes de los _____ años de edad.
- 18.0

Grupo A = Raro; Acusador, Distante, Torpe


- Paranoico
¿Cuál es el tratamiento de elección para el trastorno paranoide de la personalidad?
- Psicoterapia
- Esquizoide
(T/F) No hay un aumento de la incidencia del trastorno esquizoide de la personalidad en familias con
antecedentes de esquizofrenia.
- Verdadero
- Esquizotípico

Grupo B = salvaje; malo hasta la médula


- Antisocial
¿Qué forma de psicoterapia ha sido especialmente inútil en pacientes con trastorno antisocial de la
personalidad?
- Psicoterapia psicodinámica
 Esta nunca será la respuesta correcta en el test - encajaría en distimia o trastornos de
personalidad, trastornos de adaptación, TAG. Esta es la terapia orientada al insight; de
vez en cuando MDD.
El desarrollo de un trastorno _____ es más frecuente en pacientes con trastorno antisocial de la
personalidad a medida que envejecen.
- Somatización
¿Qué 2 criterios relacionados con la edad deben cumplirse para el diagnóstico del trastorno antisocial de la
personalidad?
- Patrón de desprecio hacia los demás y violaciones de los derechos de los demás desde los 15 años;
los pacientes deben tener al menos 18 años para el diagnóstico.
¿Qué tipo de terapia se recomienda para tratar los trastornos antisocial y límite de la personalidad?
- Terapia dialéctica conductual (TDC)

- Frontera
¿Qué trastorno de la personalidad se da hasta en el 70% de los pacientes con trastorno de identidad
disociativo?
- Trastorno límite de la personalidad
¿Qué trastorno de la personalidad se presenta con experiencias psicóticas transitorias relacionadas con el
estrés?
- Trastorno límite de la personalidad

- Histriónico
¿Qué mecanismo de defensa suelen emplear los pacientes histriónicos?
- Regresión
- Narcisista

Grupo C = preocupado; cobarde, compulsivo, pegajoso


- Evasivo
- Dependiente
- Trastorno obsesivo compulsivo de la personalidad
- Pasivo Agresivo

TRASTORNOS COGNITIVOS
Nombra 3 trastornos cognitivos principales.
- Delirio
- Demencia
o Enfermedad de Alzheimer
o Demencia vascular
o Complejo de demencia del sida
- trastorno amnésico
o Wernicke-Korsakoff
Delirio
Describa la nemotecnia AEIOU TIPSS para las causas de delirio.
- Toxicidad por alcohol/drogas
- Anomalía electrolítica
- Iatrogenia
- Hipoxia de oxígeno
- Uremia/encefalopatía hepática
- Trauma
- Infección
- Venenos
- Convulsiones
- Ictus
¿Qué afección se describe por delirio, taquicardia, temblor y tiromegalia?
- Tirotoxicosis
Junto con el delirio, ¿qué otros 2 síntomas suelen estar presentes en la encefalopatía hipertensiva?
- Presión arterial elevada
- papiledema
¿Qué clase de medicamentos deben evitarse en pacientes delirantes?
- Benzodiacepinas
¿Qué medicamento puede utilizarse para tratar el delirio anticolingérico causado por los ATC y algunos
neurolépticos?
- Betanechol
(T/F) Por regla general, las alucinaciones olfativas y táctiles son más frecuentes en los trastornos médicos que en las
psicosis.
- Verdadero
¿Cuáles son las 3 características de la encefalopatía de Wernicke?
- Delirio; oftalmoplejia; ataxia
¿Qué prueba es sensible para determinar la etiología del delirio?
- Electroencefalograma (EEG)
¿Qué antibiótico es a/w psicosis?
- Isoniazida
Diferenciar entre alucinaciones e ilusiones.
- Alucinaciones: percepción sensorial que se produce en ausencia de un estímulo real.
- ilusiones: percepción inexacta de estímulos sensoriales existentes
¿Qué 2 fármacos deben considerarse para calmar a los pacientes delirantes agitados?
- Haloperidol - IM, PO, IV
- antipsicóticos de segunda generación (por ejemplo, risperidona)

Demencia
¿Cuáles son las "4 A" de la demencia?
- Afasia
- Amnesia
- Apraxia
- Agnosia
Diferenciar entre agnosia y apraxia.
- Agnosia: incapacidad para reconocer o identificar objetos a pesar de una función sensorial intacta.
- apraxia: deterioro de la capacidad para realizar actividades motoras a pesar de que la función motora esté
intacta
Los cambios leves en la memoria que se producen con el envejecimiento normal se denominan _____.
- Olvido senescente benigno
¿Qué tipo de diagnóstico por imagen se utiliza para distinguir el Alzheimer de otras formas de demencia?
- Imagen PET de fluorodesoxiglucosa, que muestra un hipometabolismo temporal y parietal característico.
¿Qué es el deterioro cognitivo leve (DCL)?
- Estado prodrómico en el que cada año el 12% de los pacientes evolucionan hacia la demencia.
Nombra 4 características de la encefalopatía de Wernicke.
- Oftalmoplejía
- marcha atáxica
- nistagmo
- confusión mental
¿Qué 2 formas de demencia presentan rigidez de rueda dentada y temblor en reposo?
- Demencia por cuerpos de Lewy
- Enfermedad de Parkinson
¿Qué trastorno endocrinológico puede causar demencia reversible?
- Hipotiroidismo

Enfermedad de Alzheimer
¿Cuáles son los 3 genes que se consideran relacionados con la enfermedad de Alzheimer?
- Presenelina I; presenelina II; proteína precursora amiloide (APP)
¿Cuál es el principal gen de susceptibilidad al Alzheimer?
- Apolipoproteína e4 (APOe4)
Nombre 3 inhibidores de la colinesterasa utilizados para tratar la enfermedad de Alzheimer.
- Donepezilo; rivastigmina; galantamina
_____ es un antagonista NMDA aprobado para el tratamiento de la enfermedad de Alzheimer de moderada a grave.
- Memantina
¿Cuáles son los 3 inhibidores de la anticolinesterasa utilizados en el tratamiento de la demencia?
- Rivastigmina; galantamina; donepezilo

Demencia por cuerpos de Lewy


¿Cuáles son las 4 características de la demencia por cuerpos de Lewy?
- Depilación
- Parkinsonismo
- alucinaciones visuales
- sensibilidad a los neurolépticos
¿Cómo se puede diferenciar la demencia por cuerpos de Lewy de la demencia por enfermedad de Parkinson?
- Demencia con cuerpos de Lewy: aparición de la demencia en los 12 meses siguientes a los síntomas de
parkinsonismo.
- Demencia por enfermedad de Parkinson: demencia que comienza más de 12 meses después de los síntomas
de parkinsonismo.
¿Qué medicamento se utiliza para tratar el trastorno del comportamiento del sueño REM con demencia por cuerpos
de Lewy?
- Clonazepam (Klonopin)

Demencia frontotemporal
La demencia frontotemporal también se conoce como _____.
- Escoja la enfermedad

Enfermedad de Huntington
Describir la patología de la enfermedad de Huntington.
- Repetición trinucleotídica (CAG) en el brazo corto del cromosoma 4
¿Qué hallazgo de la resonancia magnética es característico de la enfermedad de Huntington?
- Atrofia caudada
(T/F) La demencia debida a la enfermedad de Parkinson se ve exacerbada por los medicamentos antipsicóticos.
- Verdadero
Describa la patología de la enfermedad de Parkinson.
- Pérdida de células en la sustancia negra de los ganglios basales, lo que provoca una disminución de la
dopamina y la pérdida de los tractos dopaminérgicos.

Enfermedad de Cruetzfeldt Jakob


¿Qué síntoma está presente en más del 90% de los pacientes con la enfermedad de Creutzfeldt-Jakob (ECJ)?
- Mioclonías (espasmos musculares repentinos)
¿Qué hallazgo del EEG es compatible con la enfermedad de Creutzfeldt-Jakob?
- Complejos trifásicos

Hidrocefalia normotensiva
Nombre 3 síntomas característicos de la hidrocefalia normotensiva.
- Ataxia; incontinencia urinaria; demencia

ABUSO Y DEPENDENCIA DE SUSTANCIAS


Abuso de sustancias/abandono
¿Qué 4 criterios definen el abuso de sustancias?
- Incumplimiento de las obligaciones en el trabajo, la escuela o el hogar.
- uso en situaciones peligrosas
- problemas legales recurrentes relacionados con sustancias
- consumo continuado a pesar de los problemas sociales o interpersonales debidos al consumo de la sustancia

Opiáceos
¿Qué 2 opiáceos no se detectan en un análisis de drogas estándar?
- Metadona
- oxicodona
¿Qué combinaciones de medicamentos están presentes en Vicodin y Percocet?
- Vicodin: hidrocodona/acetaminofeno
- Percocet: oxicodona/acetaminofeno
¿En qué se diferencia la intoxicación por meperidina de la de otros opiáceos?
- La meperidina produce midriasis, a diferencia de otros opioides, que producen miosis.
La combinación de meperidina e IMAO puede causar _____.
- Síndrome de la serotonina
o Sudoración, contracciones musculares, escalofríos, síntomas del SNC (confusión, desorientación,
hipomanía, aprensión, agitación, convulsiones).
¿Cuál es el peligro de administrar naloxona/naltrexona en caso de sobredosis de opiáceos?
- Puede provocar un síndrome de abstinencia grave en pacientes dependientes de opiáceos
¿Qué fármaco puede utilizarse para tratar los síntomas autonómicos experimentados durante la abstinencia de
opiáceos?
- Clonidina
-
Alcohol
Describir el efecto del alcohol sobre los receptores GABA y de glutamato.
- El alcohol activa los receptores GABA (inhibidores) e inhibe los receptores de glutamato (excitadores)
¿Qué clase de fármacos se utilizan para tratar el síndrome agudo de abstinencia alcohólica?
- Benzodiacepinas (a menudo clordiazepóxido)
¿Qué anomalía electrolítica puede predisponer al desarrollo de delirium tremens (DT)?
- Hipomagnesemia
¿Cuál es el plazo para el desarrollo de los DT?
- 48 - 72 horas
¿Qué define el consumo "de riesgo" o "excesivo" de alcohol?
- Hombres: > 4 bebidas al día o > 14 bebidas a la semana
- mujeres: > 3 copas al día o > 7 copas a la semana
Nombre 4 medicamentos utilizados para tratar la dependencia del alcohol.
- Disulfiram; naltrexona; acamprosato; topiramato
¿Cuál es la función del topiramato?
- Anticonvulsivo que potencia el GABA e inhibe los receptores de glutamato, reduciendo la ansiedad por el
alcohol.
¿Cuál es la principal ventaja del acamprosato?
- Puede utilizarse en pacientes con enfermedades hepáticas
Nombra 3 características de la encefalopatía de Wernicke.
- Ataxia generalizada; confusión; nistagmo
PCP
Describir el efecto farmacológico de la PCP.
- Antagoniza los receptores de glutamato NMDA y activa las neuronas dopaminérgicas
¿Qué síntoma es patognomónico de intoxicación por PCP?
- Nistagmo rotatorio
Nombra 2 marcadores séricos a menudo elevados durante el uso de PCP.
- Creatina fosfoquinasa (CPK)
- aspartato aminotransferasa (AST)
¿Qué 3 benzodiacepinas están indicadas en el tratamiento de la intoxicación por PCP?
- Diazepam; midazolam; lorazepam
¿Por qué deben evitarse los antipsicóticos típicos y las benzodiacepinas en los pacientes intoxicados por PCP?
- Antipsicóticos: los efectos secundarios anticolinérgicos pueden empeorar los efectos secundarios
- benzodiacepinas: pueden retrasar la excreción del fármaco

BZD/Barbitúricos
Diferenciar entre los efectos de las benzodiacepinas y los barbitúricos en la apertura del canal de cloruro.
- Barbitúricos: aumentan la duración de la apertura del canal de cloruro
- benzodiacepinas: aumentan la frecuencia de apertura del canal de cloruro
¿Qué tipo de síndrome de abstinencia tiene la tasa de mortalidad más alta?
- Barbitúricos
¿Qué antagonista de las benzodiacepinas de acción corta puede utilizarse para tratar la sobredosis de
benzodiacepinas?
- Flumazenil
¿Cuál es el tratamiento de elección para la sobredosis de opiáceos?
- Naloxona
Describir el tratamiento de la sobredosis de barbitúricos.
- Alcalinizar la orina con bicarbonato sódico para favorecer la excreción renal.
Nombra 3 benzodiacepinas que no son eliminadas por el hígado y que pueden usarse en pacientes con alcoholismo
crónico o enfermedad hepática.
- Lorazepam, oxazepam, temazepam; LOT
Nombre 2 benzodiacepinas de acción prolongada (> 20 horas).
- Diazepam; clonazepam
Evitar clonazepam en pacientes con _____.
- Disfunción renal
¿Cuáles son las 4 benzodiacepinas de acción intermedia (6 - 20 horas)?
- Alprazolam; lorazepam; oxazepam; temazepam
Nombre 2 benzodiacepinas de acción corta (< 6 horas).
- Triazolam; midazolam

Marihuana
¿Cuál es el efecto fisiológico del THC en el cerebro?
- Activación de los receptores cannabinoides, que inhiben la adenilato ciclasa.

Estimulantes
¿Cuál es el efecto farmacológico de la cocaína?
- Bloquean la recaptación de dopamina de la hendidura sináptica, provocando un efecto estimulante.
Describir el efecto farmacológico de las anfetaminas.
- Bloquean la recaptación y facilitan la liberación de dopamina y norepinefrina de las terminaciones
nerviosas, provocando un efecto estimulante.
Describir el efecto fisiológico de la cafeína.
- Antagonista de la adenosina, que provoca un aumento del AMPc y un efecto estimulante a través del
sistema dopaminérgico.
Nombre 2 medicamentos utilizados para tratar la dependencia de la nicotina.
- Vareniclina (Chantix)
- buproprión (Zyban)
¿Cuál es el mecanismo de la vareniclina?
- Agonista parcial del receptor colinérgico nicotínico (nAChR) alfa-4-beta-2 que imita la acción de la
nicotina y previene los síntomas de abstinencia.
Defina formicación.
- Sensación alucinada de que insectos o serpientes se arrastran por la piel.
- Efecto secundario común del consumo extensivo de cocaína y anfetaminas.

DORMIR
Describir los cambios en el sueño REM y no REM observados en los ancianos.
- Disminución de la latencia del sueño REM y disminución del sueño REM total; aumento de la cantidad de
sueño de estadio 1/2 y disminución de la cantidad de sueño de estadio 3/4.
Describir las vidas medias de los hipnóticos no benzodiacepínicos.
- Zaleplon < zolpidem < eszopiclona
¿Cuál es el mecanismo de acción de los hipnóticos no benzodiacepínicos?
- Unión selectiva al receptor 1 de las benzodiacepinas
¿Qué es el ramelteon?
- Agonista selectivo de la melatonina MT1/MT2
Diferenciar entre disomnias y parasomnias.
- Disomnias: insuficiencia, exceso o alteración del ritmo del sueño.
- parasomnias: comportamiento inusual relacionado con el sueño
¿Cuál es la diferencia temporal entre el insomnio agudo y el crónico?
- Agudo: 1 mes
¿Qué se considera tratamiento de primera línea para el insomnio crónico?
- CBT
Nombre los 3 "fármacos Z" utilizados para tratar el insomnio primario.
- Zolpidem (Ambien); eszopiclona (Lunesta); zaleplon (Sonata)
Nombre 3 antidepresivos utilizados para tratar el insomnio primario.
- Trazodona (más común); amitriptilina; doxepina
¿Cuáles son los 3 factores de riesgo de la apnea obstructiva del sueño?
- Obesidad
- aumento de la circunferencia del cuello
- estrechamiento de las vías respiratorias
Describa la tétrada clásica con narcolepsia.
- Somnolencia diurna excesiva ("ataques de sueño")
- Fenómeno del sueño relacionado con la fase REM, incluida la incapacidad para moverse durante la
transición del sueño a la vigilia.
- alucinaciones hipnagógicas/hipnopómpicas
- cataplexia
¿Qué es la cataplejía?
- Pérdida súbita del tono muscular evocada por una emoción fuerte sin LOC.
¿Cuál es el fármaco de elección para tratar la cataplejía en pacientes narcolépticos?
- Oxibato de sodio
Defina el síndrome de Kleine-Levin.
- Trastorno poco frecuente caracterizado por hipersomnia recurrente con episodios de somnolencia diurna
con hiperfagia, hipersexualidad y agresividad.
Nombra 4 trastornos del ritmo circadiano del sueño.
- Trastorno de la fase retardada del sueño
- trastorno avanzado de la fase del sueño
- trastorno del trabajo por turnos
- trastorno por desfase horario
Describa el entrenamiento en ensayo de imágenes (IRT).
- Implica el uso de imágenes mentales para modificar el resultado de una pesadilla recurrente, escribir el
resultado mejorado y luego ensayarlo mentalmente en un estado relajado.
Trastorno del sueño REM
Caracterizar el trastorno de conducta del sueño REM.
- Se caracteriza por atonía muscular durante el sueño REM y actividad motora compleja durante el sueño
mental (representación de sueños).
¿Qué 2 tipos de demencia son a/w trastorno de conducta del sueño REM?
- Atrofia olivopontocerebelosa
- Demencia por cuerpos de Lewy
¿Qué benzodiacepina es eficaz en el 90% de los pacientes con trastorno de conducta del sueño REM?
- Clonazapam

RANDOMNESS
¿Qué 4 trastornos disociativos se describen en el DSM-IV?
- Amnesia disociativa
- fuga disociativa
- trastorno de identidad disociativo
- trastorno de despersonalización

¿Qué trastorno tiene el peor pronóstico de todos los trastornos disociativos?


- Trastorno de identidad disociativo

¿Cuáles son los 3 criterios del trastorno delirante?


- Delirios fijos no extraños durante al menos 1 mes
- no cumple los criterios de la esquizofrenia
- funcionamiento en la vida no gravemente deteriorado

¿Qué es el mutismo selectivo?


- Enfermedad poco frecuente caracterizada por la negativa a hablar en determinadas situaciones (por
ejemplo, en la escuela) durante al menos 1 mes a pesar de la capacidad de comprender y utilizar el
lenguaje.

¿Qué es el síndrome de Ganser?


- Dar respuestas aproximadas a preguntas sencillas

¿Qué es el ataque de nervios?


- Trastorno de trance cultural común en Puerto Rico que consiste en movimientos convulsivos, desmayos,
llanto y problemas visuales.

¿Cuáles son las 5 etapas de la respuesta sexual normal?


- Deseo; excitación; meseta; orgasmo; resolución

¿Cuáles son las 3 parafilias más comunes?


- Pedofilia; voyeurismo; exhibicionismo

El factor más importante para evaluar el riesgo de violencia de un paciente es _____.


- Historial de violencia del individuo

Describir la formación de reacciones.


- Mecanismo de defensa mediante el cual una persona se enfrenta a un conflicto emocional o a factores
estresantes sustituyéndolos por comportamientos, pensamientos o sentimientos diametralmente opuestos a
sus propios pensamientos o sentimientos inaceptables.

Reacciones adversas a los medicamentos


¿Cuáles son los efectos secundarios de la HAM?
- antihistamínicos (sedación, aumento de peso)
- antiadrenérgicos (hipotensión)
- antimuscarínicos (sequedad de boca, visión borrosa, retención urinaria)
¿Qué 2 clases de medicamentos psiquiátricos tienen efectos secundarios?
- ATC; antipsicóticos de baja potencia
Efectos secundarios extrapiramidales
¿Cuál es el fármaco de elección para tratar el EPS producido por neurolépticos?
- Benztropina
Describa 3 formas de EPS.
- Parkinsonismo (cara de máscara, rigidez de rueda dentada, temblor de pastilla)
- acatisia (inquietud, agitación)
- distonía (contracción sostenida de los músculos del cuello, lengua, ojos, diafragma)
Nombra 3 medicamentos que pueden usarse para tratar el EPS.
- Benztropina
- Difenhidramina
- amantadina
¿Qué escala se utiliza para cuantificar y controlar la discinesia tardía?
- Escala de movimientos involuntarios anormales (AIMS)
¿Qué medicamentos pueden utilizarse para tratar la acatisia?
- Propranolol

Síndrome neuroléptico maligno


¿Cuáles son 2 rasgos característicos del síndrome neuroléptico maligno?
- Elevación de la creatina fosfoquinasa (CPK)
- "rigidez "tubería de plomo
¿Qué 2 medicamentos se utilizan habitualmente para tratar el NMS?
- Dantroleno
- bromocriptina

Síndrome de la serotonina
Describir la excitabilidad neuromuscular y el síndrome serotoninérgico.
- Hiperreflexia; movimientos de las extremidades con "sacudidas eléctricas".
¿Qué clase de medicamentos pueden ser útiles en el tratamiento del síndrome serotoninérgico?
- Antagonistas del calcio (por ejemplo, nifedipino)
Esperar al menos _____ antes de cambiar de ISRS a IMAO, y al menos _____ con fluoxetina.
- 2 semanas; 5 - 6 semanas

Interacciones medicamentosas
Nombre 4 inductores importantes de la enzima CYP450.
- Fumar (1A2)
- carbamazepina (1A2, 2C9, 3A4)
- barbituatos (2C9)
- Hierba de San Juan (2C19, 3A4)
¿Cuáles son los 5 inhibidores más importantes de la enzima CYP450?
- Fluvoxamina (1A2, 2D6, 3A4)
- fluoxetina (2C19, 2C9, 2D6)
- paroxetina (2D6)
- duloxetina (2D6)
- sertralina (2C19)

ISRS
¿Qué ISRS dispone de una forma de dosificación semanal?
- Fluoxetina
¿Qué ISRS tiene mayor riesgo de alteraciones gastrointestinales?
- Sertralina
Nombra 7 ISRS.
- Fluoxetina; sertralina; paroxetina; fluvoxamina; citalopram; escitalopram
¿Qué ISRS sólo está aprobado actualmente para el tratamiento del TOC?
- Fluvoxamina
¿Qué ISRS se prefiere en el tratamiento del trastorno explosivo intermitente?
- Fluoxetina
¿A qué neurotransmisor se atribuyen los niveles bajos de impulsividad y agresividad?
- Serotonina
¿Qué clase de efectos secundarios relacionados con los ISRS pueden aparecer más adelante en el curso del
tratamiento?
- Disfunción sexual - libido escasa, anorgasmia

IRSN
Nombra 2 IRSN.
- Venlafaxina; duloxetina
¿Cuáles son las 3 indicaciones clínicas de la venlafaxina?
- Depresión
- trastornos de ansiedad (por ejemplo, TAG)
- TDAH
La duloxetina se utiliza a menudo para pacientes con depresión y _____.
- Dolor neuropático
¿Qué contraindicación tiene la venlafaxina?
- Puede aumentar la PA; no usar en pacientes con HTA lábil no tratada.

NDRI
Describir el mecanismo del bupropión.
- Inhibidor de la recaptación de norepinefrina-dopamina
"Es tan estimulante que he tenido pacientes que han tenido orgasmos espontáneos con él".
Estimulante, umbral convulsivo.

Otros antidepresivos
¿Cuáles son las 3 indicaciones clínicas de la trazodona y la nefazodona?
- Depresión mayor refractaria; depresión mayor con ansiedad; insomnio
¿Qué efecto adverso importante tiene la trazodona? ¿Nefazodona?
- Trazodona: priaprismo
- nefazodona: insuficiencia hepática
Describir el mecanismo de la mirtazapina.
- Antagonista del receptor α2-adrenérgico
o primer IRSN en entrar en el mercado - a diferencia del Prozac - leve aumento de peso, sedación
o todos los ISRS y todos los IRSN -anfetaminas, topiramato y bupropión- no provocan aumento de
peso.

Antidepresivos tricíclicos
Describir el mecanismo de los antidepresivos tricíclicos.
- Inhiben la recaptación de norepinefrina y serotonina, aumentando la disponibilidad de monoaminas en la
sinapsis.
Nombra 7 antidepresivos tricíclicos y sus subcategorías.
- Aminas terciarias: amitriptilina, imipramina, clomipramina, doxepina; aminas secundarias: nortriptilina,
desipramina.
¿Qué ATC puede utilizarse para tratar el insomnio?
- Doxepina
¿Cuáles son las 2 indicaciones clínicas para el uso de imipramina?
- Enuresis; trastorno de pánico
¿Qué ATC tiene menos efectos secundarios anticolinérgicos?
- Desipramina
¿Qué ATC es menos probable que cause hipotensión ortostática?
- Nortriptilina
La base del tratamiento de la sobredosis de ATC es _____.
- Bicarbonato sódico intravenoso
Nombra 2 antidepresivos tetracíclicos.
- Amoxapina; maprotilina
¿Cuáles son las "3 C" de los efectos secundarios de los ATC?
- Cardiotoxicidad
- Convulsiones
- Coma
¿Qué antidepresivo tricíclico es preferible utilizar en la población anciana?
- Nortriptilina, ya que tiene menos efectos secundarios
ATC - contraindicación absoluta del ATC es el glaucoma - puede exacerbar el ángulo cerrado - disminuye la
reabsorción; 10-15% de los glaucomas son de ángulo estrecho.

Inhibidores de la monoaminooxidasa
Describir el mecanismo de los IMAO.
- Previenen la inactivación de aminas biógenas como la norepinefrina, la serotonina, la dopamina y la
tiramina mediante la inhibición irreversible de las enzimas MAO-A/B.
Describir la selectividad de las enzimas MAO-A y MAO-B.
- La MAO-A desactiva preferentemente la serotonina y la MAO-B desactiva preferentemente la
norepinefrina; ambos tipos actúan también sobre la dopamina y la tiramina.
Los IMAO se consideran más eficaces en el tratamiento de la depresión _____, caracterizada por hipersomnia,
aumento del apetito y mayor sensibilidad al rechazo interpersonal.
- Atípico
Nombre 3 IMAO.
- Fenelzina; tranilcipromina; isocarboxazida
¿Cuál es el efecto secundario más frecuente de los IMAO?
- Hipotensión ortostática

Antipsicóticos
Quetiapina - sedante
Nombra 6 medicamentos antipsicóticos atípicos.
- Clozapina; risperidona; quetiapina; olanzapina; ziprasidona; aripiprazol
_____ es el único antipsicótico que ha demostrado disminuir el riesgo de suicidio.
- Clozapina
(T/F) El 30% de las psicosis resistentes al tratamiento responden a la clozapina.
- verdadero
Diferenciar entre el mecanismo de los antipsicóticos típicos y atípicos.
- Típicos: bloquean los receptores de dopamina (D2); atípicos: bloquean tanto los receptores de dopamina
(D2) como los de serotonina (2A).
(T/F) Los antipsicóticos atípicos pueden ser más eficaces en el tratamiento de los sx psicóticos negativos, como el
aplanamiento del afecto y el retraimiento social.
- verdadero
Nombra 2 medicamentos antipsicóticos típicos de baja potencia y los efectos secundarios relacionados con el
pigmento a/w cada uno.
- Clorpromazina (coloración azulada de la piel)
- tioridazina (retinosis pigmentaria)
_____ puede utilizarse para tratar el hipo intratable.
- Clorpromazina
¿Por qué los antipsicóticos típicos de baja potencia son más letales en sobredosis que los antipsicóticos típicos de
media y alta potencia?
- Prolongación del QTc y posibilidad de bloqueo cardíaco y taquicardia ventricular
Nombre 4 antipsicóticos típicos de potencia media.
- Loxapina; tiotixeno; trifluoperazina; perfenazina
Nombre 3 antipsicóticos típicos de alta potencia - desarrollados para el tratamiento de la psicosis
- Haloperidol; flufenazina; pimozida
¿Qué 2 medicamentos antipsicóticos de primera generación presentan anomalías oculares?
- Tioridazina: pigmentación retiniana irreversible
- clorpromazina: depósitos en cristalino y córnea
Un riesgo de agranulocitosis, que requiere extracciones de sangre profilácticas semanales, es a/w _____.
- Clozapina
¿Qué 2 medicamentos antipsicóticos atípicos son los más a/w síndrome metabólico?
- Clozapina; olanzapina
o Olanzapina = provoca un aumento de PESO - una media de 25 libras de aumento de peso.
¿Qué antipsicótico atípico se considera "de peso neutro"?
- Ziprasidona
¿Qué antipsicótico atípico tiene niveles elevados de prolactina?
Risperidona
Diferenciar entre el mecanismo de los antipsicóticos típicos y atípicos.
- Típicos: bloquean los receptores de dopamina (D2)
- atípicos: bloquean tanto los receptores de dopamina (D2) como los de serotonina (2A)
(T/F) Los antipsicóticos atípicos pueden ser más eficaces en el tratamiento de los sx psicóticos negativos, como el
aplanamiento del afecto y el retraimiento social.
- verdadero

Estabilización del estado de ánimo


Nombra 4 estabilizadores del estado de ánimo.
- Litio; ácido valproico; lamotrigina; carbamazepina
Describir los rangos terapéuticos, tóxicos y letales del litio.
- Terapéutico: 0,6 - 1,2; tóxico: > 1,5; letal: > 2,0
Litio - pruebas de función tiroidea para obtener valores basales, electrocardiograma - puede exacerbar arritmias
existentes, función renal.
CONOCER el litio porque es el patrón oro
La mayoría de las veces se empieza con litio en un paciente maníaco.
¿Cuál es el efecto del uso simultáneo de AINE con litio?
- aumento de los niveles de litio
¿Cuál es el único estabilizador del estado de ánimo aprobado por la FDA para el tratamiento del trastorno bipolar en
niños mayores de 12 años?
- Litio
Efectos secundarios del litio:
- aumento de peso, temblor, trastornos gastrointestinales (úlceras anales, lesiones labiales, N/V, cualquier
cosa intermedia), fatiga, arritmias, convulsiones, bocio/ hipotiroidismo, leucocitosis (benigna) - puede
utilizarse para aumentar el recuento de glóbulos blancos en usuarios de clozapina, coma, poliuria,
polidipsia, alopecia, sabor metálico
- Aguda: náuseas con o sin vómitos, temblor fino, poliuria, confusión leve, cambios en la onda t del
electrocardiograma, debilidad muscular.
o Después: temblor grueso, ataxia, confusión, dificultad para hablar, náuseas persistentes y diarrea.
o Después: convulsiones, hipotensión, latidos irregulares, coma.
- Crónico: DI nefrogénica, hipotiroidismo/ bocio, aumento de peso, edema, erupciones y agravamiento de
afecciones cutáneas (acné, psoriasis), leucocitosis benigna, teratógeno (anomalía de ebstein).

En los casos de trastorno bipolar en los que está contraindicado el uso de litio, ¿cuál es el fármaco de elección?
- Ácido valproico
Describir la indicación clínica para el uso de carbamazepina.
- Especialmente útil para tratar los episodios mixtos y el trastorno bipolar de ciclo rápido, y menos eficaz
para la fase depresiva.
Describir el mecanismo de la carbamazepina.
- Actúa bloqueando los canales de sodio e inhibiendo los potenciales de acción
¿Cuáles son los 2 efectos secundarios catastróficos de la carbamazepina?
- Síndrome de Stevens-Johnson
- efectos teratogénicos cuando se utiliza durante el embarazo
Describir el mecanismo de la lamotrigina.
- Eficaz en los canales de sodio que modulan el glutamato y el aspartato
La lamotrigina es más eficaz para tratar _____.
- Depresión bipolar (aunque poca eficacia para la manía aguda o la prevención de la manía)
Nombra 3 efectos secundarios del topiramato.
- Pérdida de peso; acidosis metabólica hipoclorémica sin brecha aniónica; cálculos renales.

Legal
¿Cuáles son las "4 D" de la mala praxis?
- Desviación (negligencia) del deber que fue la causa directa del daño
¿En qué 3 situaciones no es necesario el consentimiento informado para tratar a menores no emancipados?
- Atención obstétrica
- Tratamiento de ETS
- tratamiento de la drogodependencia
¿Cuáles son las "4 R" del consentimiento informado?
- Motivo del tratamiento
- Riesgos y ventajas
- Alternativas razonables
- Consecuencias del rechazo al tratamiento
¿En qué 4 condiciones se considera que un menor está emancipado?
- Autoportante
- en el ejército
- casado
- tener hijos
Diferenciar entre capacidad y competencia.
- Capacidad: término clínico y puede ser evaluada por los médicos
- competencia: término jurídico y sólo puede decidirlo un juez
(T/F) La capacidad de decisión es específica de cada tarea y puede fluctuar con el tiempo.
- verdadero
¿Qué es Parens Patriae?
- Principio jurídico que apoya el internamiento involuntario de ciudadanos que no pueden valerse por sí
mismos.
¿Qué es la prueba de M'Naghten?
- Prueba más estricta utilizada como estándar de la defensa por demencia; indica que la persona no
comprende lo que estaba haciendo o su ilicitud.
Criterios diagnósticos DSM-IV

Trastornos psicóticos
- Esquizofrenia - 2 o más durante al menos 1 mes con enfermedad total > 6 meses causando
deterioro funcional social/ocupacional significativo.
o delirios, alucinaciones, habla desorganizada, comportamiento muy desorganizado o
catatónico, síntomas negativos(anhedonia, afecto aplanado o inapropiado, alogia
(pobreza del habla), avolición (apatía), falta de atención)
 cuando escribe sobre esquizofrenia, escribe sobre 4As.
 Ambivalencia
 Alucinaciones (delirios/alucinaciones)
 Afecto aplastado o inapropiado
 Asociación (tangencialidad)
o tx 4 semanas antes de determinar la eficacia
- Trastorno psicótico breve - < 1 mes
- Esquizofreniforme - 1-6 meses
- Trastorno esquizoafectivo - episodio depresivo mayor, maníaco o mixto + delirios o
alucinaciones durante 2 semanas en ausencia de d/o sx del estado de ánimo.
o Tratamiento: antipsicóticos, estabilizadores del estado de ánimo, antidepresivos, terapia
electroconvulsiva para la depresión/manía.
- Trastorno delirante: delirios fijos no extraños durante > 1 mes, pero no cumple los criterios de la
esquizofrenia y el funcionamiento en la vida no está significativamente alterado.
- Trastorno psicótico compartido - tx: separar; psicoterapia + antipsicóticos si el sx no mejora 1-2
semanas después de la separación.
- Koro - asia - el pene se encoge y desaparecerá = muerte
- Amok - Malasia, Sudeste Asiático - arrebatos repentinos de violencia no provocados sin
recuerdo - a menudo se suicida después.
- Brain Fag - África - HA, fatiga, alteraciones visuales en estudiantes varones

Trastornos del estado de ánimo (síntomas con depresión como síntoma)


- Trastorno depresivo mayor:
o Al menos un episodio de depresión mayor: 5/9 sx con uno de depresión o anhedonia
durante al menos 2 semanas. Sueño, psicomotricidad, apetito, concentración, energía,
depresión, interés, culpabilidad, ideas suicidas
o Sin manía/hipomanía
o Con rasgos psicóticos - TEC sola (mejor cuando se necesita una respuesta rápida) o
antidepresivo + antipsicótico
- Bipolar I
o Un episodio maníaco o mixto (NO se requiere depresión para el diagnóstico)
 Manía = 3 de las siguientes durante al menos 1 semana: distracción, insomnio,
grandiosidad, fuga de ideas, actividad/agitación, habla presionada, irreflexión (o
4, con humor irritable).
 Mixto = sx de manía/depresión presente durante al menos 1 semana
- Bipolar II
o Episodios depresivos mayores recurrentes con HIPOMANÍA
 Hipomanía - al menos 4 días; sin deterioro marcado del funcionamiento social o
laboral, sin rasgos psicóticos; no requiere hospitalización.
- Trastorno distímico: 2 o más de los siguientes síntomas durante al menos 2 años (o 1 año en
niños), no sin síntomas durante > 2 meses seguidos, sin episodio depresivo mayor.
o CHASES - concentración, desesperanza, apetito (+/-), sueño (+/-), Energía baja,
autoestima baja
o NUNCA tienen rasgos psicóticos
- Depresión doble - MDD con d/o distímico durante periodos residuales. Tratamiento: terapia
cognitiva y psicoterapia orientada al insight; antidepresivos simultáneamente
- Trastorno ciclotímico: periodos alternantes de hipomanía/sexo depresivo leve-moderado.
o Numerosos periodos de sx de hipomanía/depresión durante > 2 años sin periodos de
asx > 2 meses y sin hx de episodios depresivos o maníacos mayores.
- Trastorno depresivo menor - sx depresivo que no cumple los criterios de MDD, con periodos
eutímicos (a diferencia del trastorno distímico).
- Trastorno de adaptación con estado de ánimo depresivo
- Secundario a un trastorno médico
o MSE: afecto lábil (cambios rápidos repentinos congruentes con el estado de ánimo), TP
irrelevante, TC alucinaciones visuales francas (no hipnognóticas/hipnopómpicas) tienen
una propiedad metabólica, tóxica o estructural a menos que se demuestre lo
contrario. La única excepción-alucinaciones visualesde hombres hasta las rodillas ("veo
hombres pequeños", a veces verdes) = fingimiento, Función intelectual - la
concentración es a menudo lo primero que desaparece y el resto del examen es
cuestionable.
- Secundario a un trastorno por sustancias

MSE: TP irrelevante = depresión secundaria a trastorno médico, aflojamiento y tangencialidad y


concretismo

ANSIEDAD Y TRASTORNOS DE ADAPTACIÓN


- Trastorno de pánico - al menos 1 Ataque de pánico espontáneo y recurrente seguido de al
menos 1 mes de lo siguiente: preocupación por la recurrencia, preocupación por las
implicaciones del ataque, cambio significativo en el comportamiento relacionado con el ataque.
o Ataque de pánico = miedo y malestar intensos más 4 de los siguientes síntomas que
alcanzan su punto máximo en varios minutos; remiten en 25 min. (10-20 min. máximo
para dolor torácico/pulmonar) raramente > 1 h.
 Palpitaciones
 Malestar abdominal
 Entumecimiento, Náuseas
 Miedo intenso a la muerte; sensación de fatalidad inminente.
 Asfixia, escalofríos, dolor torácico
 Sudoración, temblores, SOB
o Descartar:
 Asma
 Respiración- EPOC
 Cardíaco(MVP, arritmia, ICC)
 Drogas(cocaína, anfetaminas, cafeína, nicotina, intoxicación por alucinógenos;
abstinencia de alcohol u opiáceos).
 Endocrino(tiroides, feocromocitoma)
- Agorafobia: ansiedad por estar en lugares/situaciones de los que puede resultar difícil escapar;
las situaciones se evitan, se soportan con angustia o se afrontan sólo con compañía; sx no
explicable de otro modo. Tx: SSRI
- Fobias específicas
o Miedo excesivo persistente provocado por una situación/objeto específico cuya
exposición produce ansiedad inmediata, el paciente reconoce que el miedo es excesivo,
evita la situación siempre que sea posible y, si la persona es menor de 18 años, ha
padecido el trastorno durante al menos 6 meses.
 Tratamiento: desensibilización sistemática y psicoterapia de apoyo
 Terapia conductual directa = inundación o desensibilización
 Terapia cognitivo-conductual = no es ni lo uno ni lo otro. USO PARA LA
DEPRESIÓN
 BZD o betabloqueantes (tratamiento corto)
- Fobias sociales - igual que las específicas con miedo relacionado con el entorno social en el que
puede producirse vergüenza o humillación.
o Paroxetina = trastorno de ansiedad social
- Trastorno obsesivo compulsivo: obsesiones y compulsiones de las que la persona es consciente
de que son irracionales y excesivas; causan una marcada angustia, consumen mucho tiempo o
interfieren significativamente en el funcionamiento diario. Tx: ISRS, clomipramina, terapia
conductual (exposición y prevención de respuesta)
- TEPT: respuesta a una experiencia traumática potencialmente mortal en la que el paciente
revive el trauma, evita los recuerdos del suceso y experimenta insensibilidad emocional o
hiperactivación (dificultad para dormir, ataques de ira, respuesta de sobresalto exagerada,
dificultad para concentrarse) con síntomas presentes durante al menos un mes y que pueden
aparecer en cualquier momento después de la experiencia.
- Trastorno por estrés agudo: acontecimiento traumático importante que pone en peligro la vida
, pero síntomas de ansiedad de corta duración: el acontecimiento ocurrió hace < 1 mes y los
síntomas duran < 1 mes.
- Trastorno de ansiedad generalizada: ansiedad excesiva y persistente ante acontecimientos y
actividades cotidianas con hiperactivación durante al menos 6 meses.
o Debe asociarse a 3 o más de los siguientes síntomas: inquietud, fatiga, dificultad para
concentrarse, irritabilidad, trastornos del sueño, tensión muscular
o Tratamiento: buspar, BZD (debe reducirse lo antes posible), ISRS +/- TCC, venlafaxina
o Ansiedad flotante: no se refiere a ninguna persona, acontecimiento o actividad en
concreto.
- Trastornos de adaptación: desarrollo de síntomas emocionales o conductuales en los 2 meses
posteriores a un acontecimiento estresante pero que no pone en peligro la vida (angustia grave
superior a la que cabría esperar tras un acontecimiento de este tipo con un impacto significativo
en el funcionamiento diario y los sx NO son los del duelo.
o Sx resuelto en los 6 meses siguientes a la finalización del factor estresante
o Tratamiento: la psicoterapia de apoyo es la más útil; terapia de grupo; farmacoterapia
para los síntomas asociados.

TRASTORNOS DE LA PERSONALIDAD
Grupo A: Acusador, distante, torpe
- Paranoico - 4 o más de los siguientes:
o Sospecha sin pruebas de explotación/engaño
o preocupación por las dudas de lealtad o fiabilidad
o Reticencia a confiar en los demás
o Interpretación de comentarios benignos como amenazadores o denigrantes.
o Persistencia del rencor
o Percepción de ataques a su personaje que no son evidentes para los demás/rápido
contraataque.
o Recurrencia de sospechas sobre la fidelidad del cónyuge/amante
- Esquizoide - sin deseo de relaciones cercanas - 4 o más de los siguientes
o No disfrutar ni desear relaciones estrechas
o Generalmente elige actividades solitarias
o Poco interés en la actividad sexual
o Disfrutar de pocas actividades
o Pocos amigos íntimos o confidentes
o Indiferencia ante los elogios o las críticas
o frialdad emocional, distanciamiento o afecto plano
- Esquizotípico: pensamiento mágico, comportamiento excéntrico, procesos de pensamiento
peculiares. 5 o más de los siguientes:
o Ideas de referencia
o Creencias extrañas o pensamiento mágico, incoherentes con las normas culturales.
o Sospecha
o Afecto inapropiado o restringido
o Apariencia/comportamiento extraño o excéntrico
o Pocos amigos íntimos o confidentes
o Pensamiento o discurso extraño (vago, estereotipado)
o Ansiedad social excesiva

Grupo B: Malo
- Antisocial - patrón de desprecio y violación de los derechos de los demás desde los 15 años, y el
paciente tiene al menos 18. (R/O abuso de drogas - difícil de determinar cuál fue primero);
Comienza en la infancia como trastorno de conducta . Deben estar presentes 3 o más de los
siguientes:
o Incumplimiento de las normas sociales mediante la comisión de actos ilícitos
o Engaño/mentira reiterada/manipulación de los demás en beneficio propio.
o Impulsividad/falta de planificación
o Irritabilidad y agresividad/peleas o agresiones repetidas
o Imprudencia y desprecio por la seguridad propia y ajena
o Irresponsabilidad/incapacidad para mantener el trabajo u obligaciones financieras
o Falta de remordimiento por las acciones
- Narcisista: superioridad, necesidad de admiración; se consideran especiales y explotan a los
demás en beneficio propio, pero tienen una autoestima frágil; pueden utilizarse antidepresivos
o litio según sea necesario si existe un trastorno del estado de ánimo comórbido; y al menos 5
de los siguientes:
o Sentido exagerado de la propia importancia
o Preocupación con fantasías de dinero ilimitado, éxito, brillantez
o Creer que es "especial" o único y que sólo puede relacionarse con otras personas de alto
estatus.
o Necesita excesiva admiración
o Sentimiento de derecho
o Se aprovecha de los demás en beneficio propio
o Carece de empatía
o Envidia o cree que los demás le envidian
o Arrogante o altivo
- Histriónico - Tx: psicoterapia, farmacoterapia para sx depresivo/ansiedad según sea necesario-
búsqueda de atención, comportamiento extravagante incapaz de formar relaciones duraderas y
significativas con al menos 5 de los siguientes:
o Incómodo cuando no es el centro de atención
o Inapropiadamente seductor o provocativo
o Utiliza la apariencia física para llamar la atención sobre sí mismo
o Tiene un discurso impresionista y falto de detalles
o Expresión teatral y exagerada de las emociones
o Fácilmente influenciable por otros o por la situación
o Percibe las relaciones como más íntimas de lo que realmente son
- Limítrofe - en el límite de la neurosis y la psicosis - Tx: DBT, con farmacoterapia para tratar los
síntomas psicóticos y depresivos según sea necesario (dosis bajas de antipsicóticos o SSRI -
farmacoterapia más efectiva en BPD que en cualquier otro PD) - al menos 5 de los siguientes
deben estar presentes:
o Impulsivoen al menos 2 formas perjudiciales (gasto, actividad sexual, abuso de
sustancias)
o Mal humor/afecto inestable
o Paranoiabajo estrés: ideación paranoide transitoria relacionada con el estrés o síntomas
disociativos.
o Imagen corporal inestable
o Relaciones lábilese intensas
o Suicida- amenazas recurrentes o automutilación
o Ira inapropiada
o Vulnerableal abandono
o Vacío

Grupo C: cobarde, compulsivo, pegajoso


- Evasivo - desean compañía pero son extremadamente tímidos y se hieren con facilidad; se
aferran a las relaciones pero LENTOS para involucrarse; 4 o más de los siguientes:
o Evita las ocupaciones que implican contacto interpersonal por miedo a las críticas o al
rechazo.
o Poco dispuesto a interactuar si no está seguro de caer bien
o Cauteloso en las relaciones interpersonales
o Preocupado por ser criticado o rechazado en situaciones sociales
o Inhibido en situaciones sociales nuevas porque se siente inadecuado.
o Cree que es socialmente inepto o inferior
o Reacios a participar en nuevas actividades por miedo a pasar vergüenza.
- OCPD - egosintónico; motivado por el trabajo en sí, frente a los narcisistas que están motivados
por el estatus. preocupación por el orden, el control y el perfeccionismo a expensas de la
eficiencia, presente en la edad adulta temprana en una variedad de contextos. al menos 4 de los
siguientes:
o Preocupación por los detalles, las reglas, las listas y la organización, hasta el punto de
que se pierde el objetivo principal de la actividad.
o Perfeccionismo que va en detrimento de la realización de la tarea
o Excesiva dedicación al trabajo
o Conciencia y escrupulosidad excesivas sobre la moral y la ética.
o No delega tareas
o Incapaz de desechar objetos sin valor.
o Miserly
o Rígido y testarudo.
- Dependiente -debe manifestarse en la edad adulta temprana. aferrarse a las relaciones, pero
buscar relaciones de forma activa y agresiva; Comportamiento sumiso y aferrado con excesiva
necesidad de ser cuidado con al menos 5 de los siguientes:
o Dificultad para tomar decisiones cotidianas sin la seguridad de los demás.
o Necesita que otros asuman responsabilidades en la mayoría de las áreas de la vida
o No puede expresar su desacuerdo por miedo a perder la aprobación
o Dificultad para iniciar proyectos por falta de confianza en sí mismo.
o Se esfuerza en exceso por obtener el apoyo de los demás.
o Se siente indefenso cuando está solo
o Busca urgentemente otra relación cuando termina una
o Preocupación por el miedo a que le dejen cuidar de sí mismo.

TRASTORNOS COGNITIVOS
- Demencias corticales = deterioro del funcionamiento intelectual
o Alzheimer - deterioro de la memoria más al menos 1: apraxia, afasia, agnosia,
funcionamiento ejecutivo disminuido.
 Apraxia = no puede realizar movimientos practicados aunque el sistema motor
esté intacto
 Afasia - no puede hablar/comprender frases
 Agnosia: no se puede identificar aunque el sistema sensorial esté intacto.
o Multiinfarto = aumento gradual de la gravedad, signos neurológicos focales
 Deterioro de la memoria más al menos 1: apraxia, afasia, agnosia,
funcionamiento ejecutivo disminuido
 Cambios de personalidad (depresión, ira, desconfianza frecuentes), pero mayor
preservación general de la personalidad.
 Paranoia también común
o Picos/demencia frontotemporal - lentamente progresiva
 Afasia, apraxia, agnosia
 Los cambios de personalidad y comportamiento son más prominentes al
principio de la enfermedad que en el alzheimer. Desinhibido o más apático.
 Atrofia de los lóbulos frontotemporales, cuerpos en pico (cuerpos de inclusión
intraneuronales)
o ECJ
 Demencia rápidamente progresiva (6-12 meses) tras el inicio de los síntomas
con mioclonía. También puede presentar EPS, ataxia, ceguera cortical, atrofia
muscular y mutismo
- Demencias subcorticales: síntomas afectivos y de movimiento prominentes.
o Enfermedad de Huntington = movimiento coreiforme, 35-50 años, EA, hipertonicidad,
depresión, psicosis, repetición CAG en Ch4, IRM con atrofia caudada.
o LBD/PD = rigidez de rueda dentada, temblor en reposo
 EP: bradicinesia, temblor en reposo, rigidez de rueda dentada, expresión facial
similar a una máscara, marcha arrastrando los pies, disartria (habla anormal).
 LBD: alucinaciones visuales, alteraciones del sueño REM, síntomas de EP,
o NPH = ataxia, incontinencia urinaria, ventrículos dilatados
 Debido a traumatismo, infección, hemorragia. Tx con derivación
o Hipotiroidismo = obesidad, pelo áspero, estreñimiento, intolerancia al frío
o Carencia de vitamina B12 = disminución de la posición/sentido vibratorio, megaloblastos
o Enfermedad de Wilson = temblor, LFT anormal, anillos de Kayser-Fleischer
o Neurosífilis = disminución de la posición/sentido vibratorio, pupilas de Argyll-robertson.
- Delirio - detectar con EEG (muy sensible a la ralentización de las ondas)
o Causas del delirio = I'M DELIRIOUS
 Deteriorodel suministro de sustratos cerebrales (insuficiencia vascular -
apoplejía)
 Metabólico
 Drogas
 Endocrinopatía
 Enfermedad hepática
 Infraestructura(enfermedad estructural de las neuronas corticales)
 Insuficiencia renal
 Infección
 Oxígeno
 ITU
 Privación sensorial
o Después de cirugía cardíaca: edad, alcohol, daño cerebral, diabetes
o AVC o lesión masiva - delirio + hemiparesia u otro daño neurológico focal
o Encefalopatía por HTN: PA elevada, papiledema, delirio
o Intoxicación por drogas: pupilas dilatadas, taquicardia, delirio.
o Meningitis - rigidez nucal, fotofobia, fiebre, delirio
o Tirotoxicosis - taquicardia, temblor, delirio de tiromegalia

GERIÁTRICA
- Pseudodemencia
o Déficits cognitivos aparentes en pacientes con depresión mayor.
o El inicio es agudo, el paciente hace hincapié en los fracasos (frente a la demencia, en la
que se deleita en los logros), el aturdimiento es infrecuente (frente a la demencia, en la
que es habitual), a menudo responde "No lo sé" (frente a la confabulación en la
demencia), es consciente de los problemas (frente a la inconsciencia en la demencia).
- Duelo
o Duelo normal: culpa y tristeza, trastornos leves del sueño y pérdida de peso, ilusiones,
intentos de reanudar las actividades cotidianas/trabajo, síntomas que se resuelven en 1
año, con los peores síntomas en 2 meses.
o Aflicción anormal (depresión mayor): sentimientos de culpa o inutilidad graves,
trastornos del sueño o pérdida de peso importantes, alucinaciones o delirios, ningún
intento de reanudar las actividades, ideación suicida, persistencia > 1 año con síntomas
peores > 2 meses.

NIÑOS - tienen muchos d/o de ansiedad, no tantos d/o de humor.


- Retraso mental - Causas :
o Síndrome de Down
o Anoxia (perinatal)
o Prematuridad
o Trauma de nacimiento
o Hipotiroidismo
o Malnutrición
o Exposición a toxinas
o trauma
o X frágil
o Infecciones y toxinas TORCH
- Trastornos del aprendizaje
o Rendimiento en lectura (niños), matemáticas (niñas) o expresión escrita inferior al
esperado para la edad. Tx: clases de recuperación
- Trastornos generalizados del desarrollo
o Autismo: los niños parecen desarrollarse con normalidad hasta después de los 2 años
 Deben presentarse al menos 6 síntomas:
 al menos 2 problemas de interacción social
 al menos 1 deficiencia en la comunicación
 al menos 1 patrón repetitivo y estereotipado de comportamiento y
actividades
o Asperger
 Deterioro de la interacción social (al menos 2) y comportamientos, intereses o
actividades restringidos o esteortipados PERO COMUNICACIÓN y desarrollo
cognitivo NORMALES
o Rett
 Desarrollo prenatal y perinatal normal, desarrollo psicomotor durante los
primeros 5 meses de vida, perímetro cefálico inicial normal con disminución de
la velocidad de crecimiento cefálico entre los 5 y los 48 meses, pérdida de
habilidades manuales entre los 5 y los 30 meses con anillado y lavado de
manos, pérdida de interacción social, problemas con la marcha o los
movimientos del tronco, convulsiones, episodios cianóticos y trastornos graves
del lenguaje y del desarrollo psicomotor.
o Trastorno de desintegración infantil
 Desarrollo normal en los 2 primeros años de vida con pérdida de habilidades
previamente adquiridas en al menos dos de las siguientes áreas: lenguaje,
habilidades sociales, control de esfínteres, juego o habilidades motoras.
Además, al menos dos de los siguientes: interacción social alterada, uso del
lenguaje alterado o comportamientos e intereses restringidos, repetitivos y
estereotipados.
 MÁS COMÚN EN NIÑOS (4-8x's)
- Trastornos del comportamiento perturbador
o Trastornos de conducta - por cuenta propia
 Depresión grave por d/o de conducta en niños
 Fracasar en los colegios, robar coches - trastorno de conducta - actuar
en 2 sitios = trastornos de conducta.
 Haciendo encontrar en la escuela, muy obediente, robar coches -
depresión (actuando en un lugar = depresión)
 Violación de los derechos básicos de los demás o de las normas y reglas sociales
con al menos 3 actos de:
 Agresión hacia personas o animales
 Destrucción de bienes
 Engaño/robo
 Infracciones graves de las normas
 Tratamiento: estructura del entorno, farmacoterapia (ISRS, litio), modificación
del comportamiento.
o Trastorno de oposición desafiante: después de que se le diga que no haga algo; algo a lo
que se muestra oposición.
 al menos 6 meses de comportamiento negativista, hostil o desafiante durante
los cuales estén presentes al menos 4 de los siguientes aspectos:
 pérdida frecuente de los estribos
 discusiones con adultos
 desafiar las normas de los adultos
 molestar deliberadamente a la gente
 se molesta fácilmente
 ira y resentimiento
 rencoroso
 culpar a los demás de los errores o del mal comportamiento
 Tratamiento: modificación del comportamiento y habilidades de resolución de
problemas
- TDAH - con y sin hiperactividad - conocer las clases
o Inatento, hiperactivo-impulsivo, O de tipo combinado.
o Al menos 6 síntomas persistentes durante al menos 6 meses presentados antes de los 7
años de edad
 Falta de atención:
 Problemas para escuchar, concentrarse, prestar atención a los detalles y
organizar las tareas.
 Se distrae con facilidad y suele ser olvidadizo.
 Hiperactividad/impulsividad:
 Pronunciarse, interrumpir, inquietarse, abandonar el asiento, hablar en
exceso
- Tourette/Tic
o Mvmt o vocalizaciones involuntarias con tics que ocurren múltiples veces al día casi
todos los días durante > 1 año sin un período libre de tics > 3 meses, con inicio antes de
los 18 años y angustia y deterioro en el funcionamiento social/ocupacional.
- Eliminación
o Enuresis: micción involuntaria después de los 5 años de edad, que se produce al menos
dos veces por semana durante 3 meses o con marcada alteración.
o Encopresis: evacuación involuntaria o intencionada de heces en lugares inapropiados;
debe tener al menos 4 años y presentar episodios al menos una vez al mes durante 3
meses.
- Mutismo selectivo
o No hablar en determinadas situaciones (colegio), con inicio generalmente a los 5-6 años
y más frecuente en niñas. Presente durante al menos 2 meses
- CONOCER al niño normal - ¿Aferrarse al niño? ¿Normalmente desordenado?

DISSOCIATIVO
- Amnesia disociativa: al menos un episodio de incapacidad para recordar información personal
importante, normalmente relacionado con un acontecimiento traumático o estresante, que no
puede explicarse por un olvido ordinario, y los síntomas causan angustia o deterioro del
funcionamiento diario (no puede explicarse por otro trastorno, afección médica o consumo de
sustancias). (A menudo es incapaz de recordar el nombre, pero recuerda detalles oscuros, frente
a un paciente con demencia que recuerda el nombre, pero no los detalles).
- Fuga disociativa: viaje repentino e inesperado lejos del hogar o del trabajo, además de la
incapacidad de recordar el pasado, con confusión sobre la identidad personal o asunción de una
nueva identidad, NO debida a un trastorno de identidad disociativo ni a los efectos fisiológicos
de un trastorno por sustancias o médico. Deterioro de la fxn social o laboral.
- Trastorno de Identidad Disociativo - al menos dos o más identidades distintas que
recurrentemente toman el control del comportamiento de una persona con la incapacidad de
recordar información personal de una personalidad cuando la otra es dominante; no debido al
uso de sustancias/condición médica.
o PEOR pronóstico de todos los trastornos disociativos
- Trastorno de despersonalización
o Experiencias persistentes o recurrentes de estar desconectado del propio cuerpo o de
los procesos mentales, pero la prueba de realidad permanece intacta durante el
episodio; deterioro social u ocupacional que no puede explicarse por otro trastorno
mental o físico.

Tx: recuperar los recuerdos perdidos mediante hipnosis, amobarbital sódico o lorazepam durante la
entrevista.
PREGUNTAS PRÁCTICAS

P1) Un niño de 4 años acaba de reunirse con su madre biológica tras pasar 6 meses en acogida. Inicialmente fue
expulsado de su hogar por negligencia grave secundaria al abuso de sustancias. Se observa que el niño juega de
forma destructiva y desorganizada, tirando y rompiendo sus juguetes. Cuando la madre intenta impedir que le lance
bloques, empieza a dar patadas y a morder. ¿Cuál de los siguientes es el diagnóstico más probable?

a) Trastorno negativista desafiante (TOD)


b) TDAH
c)Trastorno reactivo del apego
d) TEPT
e) Depresión mayor

P2) ¿Cuál es el mejor ejemplo de prevención secundaria?


a) Inscribir a un niño de 2 años en un grupo de juego con la esperanza de prevenir conductas antisociales.
b)Restituir a un adolescente violento después de un intento de suicidio.
c) Visitar la casa de un niño después de que un vecino informe de que el niño está siendo desatendido.
d) Tratar sentimientos de depresión y anhidonía de nueva aparición con un ISRS.
e) Inscribir a hermanos de pacientes bipolares en terapia de grupo.

P3) Un hombre de 56 años es llevado a su consulta por su mujer porque ha notado un cambio de personalidad en los
últimos 3 meses. Mientras el paciente es entrevistado, comenta continuamente sucesos de las noticias, lo que ha
comido y otros temas no relacionados con el interrogatorio antes de dar respuestas adecuadas a sus preguntas. ¿Cuál
de los siguientes síntomas se ajusta mejor al comportamiento de este paciente?
a) Síntomas negativos
b) Desorientación
c) Pensamiento concreto
d) Perseverancia
e) Circunstancialidad

1. Una niña de 3 años es llevada al médico por sus padres


porque están preocupados por su comportamiento. Describen
su hija como testaruda y siempre en movimiento. Puede
rara vez permanecen sentados más de 10 minutos. A menudo se niega a
cumplir con sus peticiones y a veces lanza de 3 a 5-
rabietas de un minuto. Informan de que se entretiene en
acostarse y necesita indicaciones frecuentes y ayuda para
preparándose para ir a la cama. Su profesora de preescolar señala que es
activo y hablador sin ser molesto y está empezando a
para demostrar un juego más interactivo con sus compañeros. Ella
generalmente duerme toda la noche y de vez en cuando moja el
cama. Tiene buen apetito. Su primera palabra fue a los 11 años
meses, y empezó a andar sin ayuda a la edad de
de 14 meses. La exploración física no muestra anomalías.
En el examen del estado mental, inicialmente se esconde detrás de su
madre, pero se encariña con el entrevistador al cabo de unos minutos y
empieza a jugar con juguetes en la oficina. Su discurso es 90%
inteligible, y su vocabulario es amplio para su edad. ¿Cuál de
el siguiente paso es el más adecuado para
gestión?
(A) Tranquilidad
(B) Terapia de juego
(C) Logopedia
(D) Alarma de enuresis
(E) Ensayo con fluoxetina
(F) Ensayo de metilfenidato

2. Un hombre de 45 años es llevado al médico por su cónyuge.


Ha estado bebiendo en exceso desde que fue pasado por alto por un
promoción laboral hace 3 días. Se quedó en cama durante el
fin de semana. No tiene antecedentes personales de trastornos psiquiátricos
y sin antecedentes personales o familiares de abuso de alcohol. Él es
llorando y afirma: "AI no puede creerlo", cuando se le aborda. En
Preguntado qué hará, afirma: "No lo sé, pero si no lo hago...
volver a trabajar mañana, perderé mi trabajo.@ ¿Cuál de las
¿cuál es el diagnóstico más probable?
(A) Trastorno de adaptación con estado de ánimo depresivo
(B) Trastorno bipolar
(C) Trastorno distímico
(D) Trastorno depresivo mayor
(E) Abuso de sustancias

3. Una mujer de 18 años previamente sana es llevada al


médico para una evaluación debido a la pérdida de apetito,
insomnio e irritabilidad extrema durante 3 semanas. En
faltando a muchos entrenamientos, abandonó el equipo universitario de softball que
que antes disfrutaba. A menudo se siente cansada y tiene
dificultad para permanecer sentado y concentrarse en las tareas escolares. Su
La menstruación se produce a intervalos regulares. Pesa 50 kg (110 lb)
y mide 168 cm; su IMC es de 18 kg/m2. Su pulso
es de 74/min, las respiraciones son de 16/min, y la presión arterial es de
110/70 mm Hg. ¿Cuál de las siguientes opciones es la más probable
¿Diagnóstico?
(A) Trastorno de adaptación con alteración mixta de
emociones y conducta
(B) Anorexia nerviosa
(C) Trastorno por déficit de atención con hiperactividad
(D) Trastorno distímico
(E) Trastorno depresivo mayor

4. Un hombre de 57 años acude al médico acompañado de


a su mujer por un historial de fatiga de dos años. Informa
despertar cansado casi todas las mañanas, a menudo con dolor de cabeza.
Duerme la siesta casi todas las tardes. Cree que la fatiga es
afectando a su concentración y rendimiento en el trabajo. Su mujer
dice que ronca con frecuencia durante la noche y a veces
se despierta jadeando. Ella lo describe como un inquieto
durmiente. Le extirparon las amígdalas y las adenoides cuando era niño.
niño. No tiene antecedentes de enfermedad grave y no toma
medicamentos. Mide 178 cm y pesa 115 kg.
(253 lb); el IMC es de 36 kg/m2. Su pulso es de 86/min, y la sangre
es de 164/88 mm Hg. El tabique nasal se encuentra en la línea media.
La exploración no muestra otras anomalías. ¿Cuál de los
¿cuál es el diagnóstico más probable?
(A) Síndrome de fatiga crónica
(B) Narcolepsia
(C) Trastorno del comportamiento del sueño REM
(D) Síndrome de las piernas inquietas
(E) Apnea del sueño

5. Una mujer de 52 años cuyo marido murió hace 2 meses


consulta a un médico por dolores de cabeza y sensación de
incertidumbre. Describe los dolores de cabeza como una banda a su alrededor
cabeza; se producen de forma imprevisible y no van acompañadas de
cualquier otro síntoma. No tiene antecedentes de enfermedad psiquiátrica.
Mientras habla con el médico, el paciente empieza a llorar y
hablar de su difunto marido; siente que su vida está vacía
ahora y se preocupa por su futuro. ¿Cuál de las siguientes opciones es
más apropiado en este momento?
(A) Permitirle expresarse
(B) Prescribir un ansiolítico
(C) Prescribir un medicamento antidepresivo
(D) Remitirla a pruebas psicológicas
(E) Obtener una consulta psiquiátrica

6. Un hombre de 47 años es llevado al servicio de urgencias por


policía después de que fuera encontrado comiendo basura de un contenedor
detrás de un restaurante. Dice que acaba de llegar a esta ciudad y
que no tiene casa, así que no tiene dinero para comida. Admite
varias hospitalizaciones psiquiátricas en el pasado, pero dice que
ya no necesita medicación. Parece sucio y está
maloliente. Los signos vitales están dentro de los límites normales. Físico
El examen no muestra anomalías. Sobre el estado mental
examen, su discurso es claro, pero su proceso de pensamiento es
desorganizada con muchas asociaciones sueltas. En varias ocasiones
durante la entrevista, parece estar preocupado por
estímulos internos. Dice que oye voces que tienen un
conversación en curso en su cabeza. ¿Cuál de las siguientes opciones es
¿el diagnóstico más probable?
(A) Trastorno bipolar
(B) Trastorno psicótico breve
(C) Trastorno delirante
(D) Trastorno depresivo mayor con rasgos psicóticos
(E) Trastorno psicótico debido a una enfermedad médica general
condición
(F) Esquizofrenia
(G) Trastorno esquizotípico de la personalidad

7. Una mujer de 32 años es llevada al servicio de urgencias


por fiebre, alucinaciones, agitación y confusión durante 8
horas. Tiene antecedentes de consumo de alcohol, cocaína y
abuso de benzodiacepinas. Su temperatura es de 37,8°C (100°F),
El pulso es de 110/min, la respiración es de 16/min, y la presión sanguínea
es de 150/90 mm Hg. La exploración muestra temblores y
telangiectasia. Los pulmones están limpios a la auscultación. Hay un
soplo holosistólico; el abdomen está sensible, y el hígado
El borde es palpable a 3 cm por debajo del margen costal derecho. Rectal
El examen no muestra anomalías. Su suero alcalino
actividad de la fosfatasa es de 200 U/L, la actividad de la ALT en suero es de 60
U/L, y la actividad sérica de AST es de 90 U/L. ¿Cuál de las
¿Cuál es la causa más probable de esta afección?
(A) Toxicidad aguda de la cocaína
(B) Abstinencia de alcohol
(C) Abstinencia de benzodiacepinas
(D) Trastorno de pánico
(E) Trastorno esquizofreniforme
8. Un niño de 10 años es llevado al médico a causa de
aumento de los problemas de comportamiento en la escuela desde que empezó 5º
grado hace 3 meses. Su profesor afirma que es incapaz de sentarse
en silencio durante un período de clase y con frecuencia interrumpe la
clase e interrumpe a otros niños mientras hablan. Su
Los padres informan de que siempre ha sido un niño activo y son
preocupado porque no está atento cuando corre o camina.
Durante la exploración, se mueve nerviosamente con las manos y los pies y es
se distrae fácilmente de completar una tarea. ¿Cuál de los
¿cuál es la farmacoterapia más adecuada?
(A) Amitriptilina
(B) Fluoxetina
(C) Haloperidol
(D) Imipramina
(E) Metilfenidato

9. Una mujer de 27 años es llevada al servicio de urgencias


1 hora después de que un amigo la encontrara apenas excitable en su
apartamento desordenado con un gato casi muerto de hambre. El paciente
parece extremadamente delgada. Su pulso es de 90/min, las respiraciones son
6/min, y la tensión arterial es de 90/60 mm Hg. Físico
el examen muestra pupilas pequeñas, labios agrietados y hematomas y
arañazos en las extremidades superiores. Estado mental
el examen muestra obtundación leve, afecto embotado y
habla lenta e incoherente. ¿Cuál de las siguientes es la
¿el siguiente paso adecuado en la gestión?
(A) Observación en una habitación tranquila y oscura
(B) Administración oral de clorpromazina
(C) Administración intramuscular de naloxona
(D) Administración intravenosa de haloperidol
(E) Administración intravenosa de lorazepam

10. Un niño sano de 9 años es llevado al médico por su


padres porque les preocupa que no le guste ir a clase.
escuela. Cada mañana llora y suplica quedarse en casa. Él
falta al colegio al menos 1 día a la semana porque su madre está
agotado de luchar con él para asistir. Sus profesores
informan de que está callado en clase y rara vez participa. Tiene
dificultad para leer al nivel de sus compañeros y carece de
confianza. En casa, tiende a permanecer en la misma habitación que su
madre y a veces la sigue por toda la casa.
Cuando sus padres planean una salida nocturna, a menudo se convierte en
llorosa y hace muchas preguntas sobre cuándo volverán.
Le gusta invitar amigos a su casa y parece que disfruta
estar con ellos. El examen físico no muestra
anomalías. Durante el examen, se sienta en el
regazo y es tranquilo pero cooperativo. Hace un breve contacto visual
y habla en voz baja, poniéndose llorosa cuando
cuestionado por estar lejos de su madre. ¿Cuál de los
¿cuál es el diagnóstico más probable?
(A) Trastorno distímico
(B) Retraso mental leve
(C) Trastorno negativista desafiante
(D) Trastorno de la lectura
(E) Trastorno de ansiedad por separación
(F) Fobia social
11. Una mujer de 47 años es llevada al médico por su
marido por comportamiento extraño durante 1 semana. Su marido
dice que no tiene sentido cuando habla y parece estar
viendo cosas. También ha tenido dificultades para dormir durante 2
meses y ha ganado aproximadamente 9 kg (20 lb) durante el
últimos 5 meses. Durante este tiempo, ha estado de mal humor y
se fatiga fácilmente. También observa que la forma de su cara tiene
se vuelven cada vez más redondos y desproporcionados con el resto
de su cuerpo a pesar de su aumento de peso. No tiene antecedentes de
enfermedad psiquiátrica o médica. Mide 160 cm.
y pesa 70 kg (155 lb); su IMC es de 28 kg/m2. Su pulso es
98/min, respiraciones 8/min, y presión sanguínea 148/92
mm Hg. La exploración física muestra obesidad troncular y
Equimosis en las extremidades superiores e inferiores. Neurológico
El examen no muestra hallazgos focales. Estado mental
El examen muestra un discurso presionado y desorganizado.
proceso de pensamiento. Hay pruebas visuales y auditivas
alucinaciones. El análisis toxicológico de orina es negativo. Qué
¿Cuál de los siguientes es el diagnóstico más probable?
(A) Trastorno psicótico breve
(B) Trastorno depresivo mayor con rasgos psicóticos
(C) Trastorno psicótico debido a una enfermedad médica general
condición
(D) Esquizofrenia
(E) Trastorno esquizotípico de la personalidad

12. Un día después de ingresar en el hospital por agitación y


alucinaciones, un hombre de 19 años tiene la aparición de graves
rigidez muscular que le impide levantarse de la cama. En
en el momento del ingreso, se inició tratamiento con haloperidol.
Hoy, parece letárgico y diaforético. Su temperatura
es de 39,7°C (103,5°F), el pulso es de 120/min y la tensión arterial es de
160/110 mm Hg. El examen físico muestra
Rigidez severa de las extremidades superiores bilateralmente. En mental
examen de estado, no está orientado a persona, lugar y
tiempo. ¿Cuál de los siguientes pasos es el más adecuado?
en la gestión?
(A) Sólo observación
(B) Añadir fluoxetina
(C) Añadir carbonato de litio
(D) Interrumpir el haloperidol
(E) Aumentar la dosis de haloperidol.

13. Una mujer de 32 años acude al médico a causa de un 3-


Historia de una semana de estado de ánimo depresivo. Trabaja en las noticias locales
ancla. Dice que siempre ha tenido una agenda muy apretada, pero
últimamente no ha tenido su habitual cantidad de energía y ha tenido
dificultad para levantarse e ir a trabajar. Se describe a sí misma
como normalmente una persona "hiper" con energía para realizar múltiples
tareas. Durante los últimos 10 años, ha tenido episodios similares
en la que ha tenido un estado de ánimo depresivo asociado a una
Disminución del nivel de energía que la hace sentirse "ralentizada".
Los episodios nunca duran más de unas semanas. Ella
a veces pasa por periodos en los que siente una oleada de
energía, duerme muy poco, se siente en la cima de su mentalidad
y es capaz de generar nuevas ideas para el canal de noticias;
estos episodios nunca duran más de 5 días. Dice que
le encanta sentirse así y desea que los episodios duren
más larga. No toma medicamentos. No bebe alcohol
o consumir drogas ilícitas. Su temperatura es de 37°C (98,6°F), su pulso es de
70/min, y la tensión arterial es de 125/80 mm Hg. Físico
El examen no muestra anomalías. Estado mental
el examen muestra un estado de ánimo deprimido y un afecto plano. Qué
¿Cuál de los siguientes es el diagnóstico más probable?
(A) Trastorno por déficit de atención con hiperactividad
(B) Trastorno ciclotímico
(C) Trastorno distímico
(D) Trastorno depresivo mayor
(E) Trastorno del estado de ánimo debido a una afección médica general

14. Una mujer de 77 años es llevada al servicio de urgencias


por su marido a causa de agitación y confusión durante 3
horas. Afirma que ha estado gritando intermitentemente
y no parece reconocerle. Una rutina sanitaria
el examen de mantenimiento de hace 3 días no mostró anormalidades
excepto por déficits leves de memoria. Su temperatura actual es
37,8°C (100°F), pulso de 100/min, respiraciones de 14/min, y
La tensión arterial es de 130/60 mm Hg. El examen físico muestra
ninguna anormalidad excepto una leve sensibilidad a la palpación del
bajo vientre. El examen del estado mental muestra confusión;
está orientada a la persona, pero no al tiempo ni al lugar. ¿Cuál de los
¿cuál es el siguiente paso más adecuado en el diagnóstico?
(A) Determinación de la velocidad de sedimentación globular
(B) Medición de la fosfatasa alcalina sérica
actividad
(C) Medición de la concentración sérica de folato
(D) Análisis de orina
(E) Ensayo Western blot

15. Un niño de 14 años es llevado al médico por su madre


después de encontrar un cigarrillo de marihuana sin fumar en su
dormitorio. La madre informa de que su hijo nunca ha hecho
nada como esto antes. Su rendimiento académico es
excelente. Cuando se le entrevista a solas, el paciente informa de que su
amigos oyeron hablar de fumar marihuana y adquirieron
de sus compañeros para saber cómo era. Añade que
nunca ha fumado marihuana. Solicita que su
profesores no fueran informados porque estarían muy
decepcionado si se enteraban. El examen físico muestra
ninguna anomalía. En el examen del estado mental, es agradable
y cooperativo y parece arrepentido. ¿Cuál de los
¿cuál es el diagnóstico más probable?
(A) Trastorno de conducta
(B) Abuso de marihuana
(C) Dependencia de la marihuana
(D) Problema relacional entre padres e hijos
(E) Adolescencia normal

16. Un hombre de 27 años, por lo demás sano, es remitido a un


cardiólogo debido a tres episodios de palpitaciones graves,
malestar torácico sordo y sensación de ahogo. Los episodios
ocurren repentinamente y se asocian con náuseas, desmayos,
temblores, sudoración y hormigueo en las extremidades; siente
como si se estuviera muriendo. A las pocas horas de cada episodio, el físico
La exploración y las pruebas de laboratorio no muestran anomalías. Él
no abusa de las drogas ni del alcohol y no tiene antecedentes de
problemas interpersonales. ¿Cuál de las siguientes es la
¿diagnóstico probable?
(A) Trastorno delirante
(B) Trastorno de ansiedad generalizada
(C) Hipocondría
(D) Trastorno de pánico
(E) Trastorno de somatización

17. Una mujer de 42 años es llevada al médico por su


marido por tristeza persistente, apatía y llanto
desde hace 2 meses. Tiene un historial de 10 años de
lupus eritematoso mal controlado con corticosteroides
terapia. La exploración física muestra un eritema de 1 cm.
lesiones sobre las extremidades superiores y el cuello y una malar
erupción de mariposa. En el examen del estado mental, parece
deprimido. Dice que estaría mejor muerta. Qué
de los siguientes es el paso siguiente más adecuado en
gestión?
(A) Preguntar a la paciente sobre sus pensamientos suicidas
(B) Tranquilizar a la paciente diciéndole que se pondrá bien.
(C) Recomendar la hospitalización psiquiátrica
(D) Iniciar tratamiento con paroxetina
(E) Aumentar la dosis de corticosteroides

18. Un hombre de 27 años es llevado al servicio de urgencias por


policía 2 horas después de amenazar a su vecino de al lado. En
vecino llamó a la policía tras recibir una nota exigiendo
que dejara de grabar en vídeo todas las actividades de la paciente.
casa o llamaría a la policía. Durante el examen, el
El paciente coopera. Explica que ha vivido en el
barrio durante 8 meses. Hace tres meses, se dio cuenta de que
su vecino instaló una nueva antena parabólica y dice que desde que
ese tiempo, ella ha estado observando cada movimiento que él hace. Él
no tiene antecedentes personales ni familiares de enfermedad psiquiátrica. Él
no ha tenido cambios en el patrón de sueño y rinde bien en su
trabajo como vendedor de coches. Aparece pulcramente vestido. Físico
El examen no muestra anomalías. Sobre el estado mental
examen, su proceso de pensamiento es organizado y lógico.
No hay evidencia de ideación suicida u homicida o
alucinaciones. Dice que no sospecha de nadie
que no sea su vecino. ¿Cuál de las siguientes es la
¿diagnóstico probable?
(A) Trastorno bipolar
(B) Trastorno psicótico breve
(C) Trastorno delirante
(D) Trastorno depresivo mayor con rasgos psicóticos
(E) Esquizofrenia

19. Una niña de 9 años es llevada al médico por su adoptante


padres porque están preocupados por su creciente
dificultades en la escuela desde que empezó tercero hace 7 semanas.
Sus profesores afirman que se frustra con facilidad y que ha tenido
dificultad para leer y prestar atención. También ha tenido
mayor impulsividad y más dificultad de lo habitual para hacer
y conservar amigos. Su madre biológica abusó de múltiples
sustancias antes y durante el embarazo, y la paciente fue
adoptada poco después de nacer. Se encuentra en el percentil 20 para
de altura y el percentil 40 de peso. El examen muestra un
puente nasal aplanado y un largo filtrum. Durante el
examen, es alegre. Las pruebas psicopedagógicas muestran
un coeficiente intelectual de 82. La explicación más probable de estos resultados es
¿exposición in utero a cuál de las siguientes?
(A) Alcohol
(B) Cocaína
(C) Éxtasis (3,4-metilendioximetanfetamina)
(D) Heroína
(E) Marihuana
(F) PCP (fenciclidina)
(G) Tolueno

20. Un hombre de 77 años acude al médico con su hija


para un examen de seguimiento con el fin de conocer los resultados de
pruebas neuropsicológicas realizadas hace 1 semana para
evaluación de una pérdida de memoria reciente. Resultados de las pruebas
indicaron cambios cognitivos consistentes con las primeras etapas de
demencia. Hace tres semanas, recibió el diagnóstico de
cáncer de próstata y ha mostrado signos de depresión
desde entonces. Hace veinte años, necesitó tratamiento en un
hospital por trastorno depresivo mayor. Sus síntomas se resolvieron
con terapia antidepresiva, y no ha tomado ninguna
medicación psicotrópica durante los últimos 15 años. El paciente
la hija entra en la sala de reconocimiento antes que su padre
y pide que el médico no le diga a su padre ninguna información
que podría ser molesto, dada su vulnerabilidad a la depresión.
Dice que le preocupan los resultados que puedan obtenerse y
cómo los manejará su padre. El paciente entra en la habitación
poco después de que su hija haga su petición. ¿Cuál de los
la siguiente es la declaración inicial del médico más adecuada para
¿Este paciente?
(A) "Debido a su historial de depresión, me gustaría
me gustaría que empezaras a tomar una medicación antidepresiva
antes de seguir hablando de su
pruebas neuropsicológicas".
(B) "Antes de repasar los resultados de sus pruebas, me gustaría
oír cómo te ha ido. Usted ha sido
en un momento difícil".
(C) "Me gustaría hablar un poco con su hija sobre
los resultados de tus pruebas, y luego repasaré las cosas
contigo".
(D) "Su hija está preocupada por usted. Creo que
necesitas ver a un psiquiatra antes de que vayamos a ninguna
más aquí".
(E) "Sus pruebas no fueron concluyentes, y me gustaría
que empiece a tomar una medicación que le ayude con su
memoria como precaución".
1. Una chica de 17 años, previamente sana, es llevada al médico para evaluación debido a pérdida de apetito,
insomnio y extrema irritabilidad durante 3 semanas. Tras faltar a muchos entrenamientos, abandonó el equipo de
softball del que antes disfrutaba. A menudo se siente cansada y tiene dificultades para permanecer sentada y
concentrarse en las tareas escolares. Su menstruación se produce a intervalos regulares. Pesa 50 kg y mide 168 cm.
Su tensión arterial es de 110/70 mm Hg, su pulso de 74/min y su respiración de 16/min. ¿Cuál de los siguientes es el
diagnóstico más probable?
(A) Trastorno de adaptación con alteración mixta de las emociones y la conducta.
(B) Anorexia nerviosa
(C) Trastorno por déficit de atención con hiperactividad
(D) Trastorno distímico
(E) Trastorno depresivo mayor

2. Un hombre de 45 años es llevado al médico por su cónyuge. Ha estado bebiendo mucho desde que le rechazaron
para un ascenso laboral hace 3 días. Se quedó en cama el fin de semana. No tiene antecedentes personales de
trastornos psiquiátricos ni antecedentes personales o familiares de abuso de alcohol. Está llorando y afirma: "No me
lo puedo creer", cuando se le aborda. Cuando se le pregunta qué hará, afirma: "No lo sé, pero si no vuelvo a trabajar
mañana, perderé mi empleo". ¿Cuál de los siguientes es el diagnóstico más probable?
(A) Trastorno de adaptación con estado de ánimo depresivo
(B) Trastorno bipolar
(C) Trastorno distímico
(D) Trastorno depresivo mayor
(E) Abuso de sustancias

3. Una mujer de 52 años cuyo marido murió hace 2 meses consulta a un médico debido a dolores de cabeza y
sentimientos de incertidumbre. Describe los dolores de cabeza como una banda alrededor de la cabeza; se producen
de forma imprevisible y no van acompañados de ningún otro síntoma. No tiene antecedentes de enfermedad
psiquiátrica. Mientras habla con el médico, la paciente empieza a llorar y a hablar de su marido fallecido; siente que
su vida está vacía ahora y se preocupa por su futuro. ¿Cuál de las siguientes opciones es la más adecuada en este
momento?
(A) Permitirle expresarse
(B) Prescribir un ansiolítico
(C) Prescribir un medicamento antidepresivo
(D) Remitirla a pruebas psicológicas
(E) Obtener una consulta psiquiátrica

4. Un niño de 10 años es llevado al médico debido a crecientes problemas de conducta en la escuela desde que
empezó 5º curso hace 3 meses. Su profesor afirma que es incapaz de permanecer sentado en silencio durante un
periodo de clase y que con frecuencia altera la clase e interrumpe a otros niños mientras hablan. Sus padres informan
de que siempre ha sido un niño activo y están preocupados porque no está atento cuando corre o camina. Durante el
examen, se mueve nerviosamente con las manos y los pies y se distrae con facilidad para completar una tarea. ¿Cuál
de las siguientes opciones es la farmacoterapia más adecuada?
(A) Amitriptilina
(B) Fluoxetina
(C) Haloperidol
(D) Imipramina
(E) Metilfenidato

5. Una mujer de 32 años es llevada al servicio de urgencias por fiebre, alucinaciones, agitación y confusión durante
8 horas. Tiene antecedentes de abuso de alcohol, cocaína y benzodiacepinas. Su temperatura es de 37,8 C (100 F), la
tensión arterial de 150/90 mm Hg, el pulso de 110/min y la respiración de 16/min. Está temblorosa. Los pulmones
están limpios a la auscultación. Presenta un soplo holosistólico; el abdomen está sensible y el borde hepático es
palpable 3 cm por debajo del margen costal. El tacto rectal no muestra anomalías. Tiene telangiectasia. Un
hemograma completo y pruebas de función hepática no muestran anomalías. Su actividad sérica de fosfatasa alcalina
es de 200 U/L, la actividad sérica de alanina aminotransferasa (ALT, GPT) es de 60 U/L, y la actividad sérica de
aspartato aminotransferasa (AST, GOT) es de 90 U/L. ¿Cuál de las siguientes es la causa más probable de esta
afección?
(A) Toxicidad aguda de la cocaína
(B) Abstinencia de alcohol
(C) Abstinencia de benzodiacepinas
(D) Trastorno de pánico
(E) Trastorno esquizofreniforme

Respuestas a la primera serie de preguntas:


A1) c - El trastorno reactivo del apego es el resultado de una relación temprana gravemente disfuncional entre el
cuidador principal y el niño. Cuando los cuidadores desatienden sistemáticamente las necesidades físicas o
emocionales del niño, éste no consigue desarrollar un apego seguro y estable con ellos. Este fracaso provoca una
grave alteración de la capacidad del niño para relacionarse con los demás, que se manifiesta en diversos problemas
de comportamiento e interpersonales. Algunos niños son temerosos, inhibidos, retraídos y apáticos; otros son
agresivos, perturbadores y desorganizados, con baja tolerancia a la frustración y escasa modulación del afecto. Este
trastorno se confunde a menudo con el TOD o el TDAH.
A2) d - Tratar los sentimientos de depresión y anhidonía de nueva aparición con un ISRS. l objetivo de la
prevención secundaria es abordar los problemas en una fase temprana para evitar futuros problemas más graves.
A3) e- La circunstancialidad es una alteración en la que el paciente divaga en detalles innecesarios antes de
comunicar la idea central.

Clave de respuestas para las preguntas de ejemplo de psiquiatría


(Preguntas 1-20)
1. A
2. A
3. E
4. E
5. A
6. F
7. B
8. E
9. C
10. E
11. C
12. D
13. B
14. D
15. E
16. D
17. A
18. C
19. A
20. B

Últimos 5 ejemplos de preguntas

1. E
2. A
3. A
4. E
5. B

Anda mungkin juga menyukai